>> Åwªï±z¡A³X«È¡G µn¤J½×¾Â «ö³o¸Ìµù¥U §Ñ°O±K½X ¦b½u·|­û ¤å³¹·j´M ½×¾Â­·®æ  ¨Ï¥Î»¡©ú   


>>> §ä¤H¿i¤ú©Î¥DÃD»Pª©­±µLÃöªº¹Ï¤å
¶ð¤º´Óª«¶é ¡÷ [ ´ú¸Õ°Ï ] [ªð¦^] ¡÷ ÂsÄý¡GSample cases¡@ ¼Ð°O½×¾Â©Ò¦³¤º®e¬°¤wŪ¨ú 

 ¥Ø«e½×¾ÂÁ`¦b½u 15 ¤H¡A¥»¥DÃD¦@¦³ 1 ¤HÂsÄý¡C¨ä¤¤µù¥U·|­û 0 ¤H¡A³X«È 1 ¤H¡C¡@ [Ãö³¬¸Ô²Ó¦W³æ]
µoªí¤@½g·s¥DÃD ¦^ÂФ峹 ¶}±Ò¤@­Ó·s§ë²¼ ¡»¦¹¤å³¹³Q¾\Ū 4770 ¦¸¡»¡@¡@ÂsÄý¤W¤@½g¥DÃD ¡@­«·s¾ã²z¥»¥DÃD  ¾ðª¬Åã¥Ü¤å³¹¡@ÂsÄý¤U¤@½g¥DÃD
 * ¤å³¹¥DÃD¡G Sample cases   ¥»¤å³¹¦³°ÝÃD¡A¶Ç°eµu°T®§³ø§iµ¹ª©¥D  ¥[¨ì§Úªº³Ì·R&Ãöª`¥»¤å³¹  Åã¥Ü¥i¦C¦Lªºª©¥»  §â¥»¤å³¹¥[¨ì§Úªº³Ì·R   
JuanFe 

 

¸ê®Æ: ¦¹·|­û¥Ø«e¤£¦b½u¤W Male ¨°Às ¥Õ¦Ï®y
«Â±æ: 0
¾y¤O: ¾y¤O: 78957
¸gÅç: ¸gÅç: 35986
¨Ó¦Û: ¦t©z¤¤¡@blank
µo¤å: 1118 ½g
ºëµØ: 0 ½g
¦b½u: 47¤Ñ19®É22¤À57¬í
µù¥U: 2013/06/17
Message ¬d¬Ý¡@·j´M¡@³q°T¿ý¡@½Æ»s¡@¤Þ¥Î¡@¦^ÂФ峹¦^ÂС@[²Ä 31 ¼Ó]
 30 Case 30 scenario ( multiple bruises )

Doorway information about patient

The patient is a 32-year-old woman who comes to the emergency department due to multiple bruises

Vital signs

. Temperature : 37.4¡¦C(99.3F)
. Blood pressure : 120/80 mmHg
. Pulse : 90/min
. Respiration : 16/min

Basic differential diagnosis

. Accident
. Physical assault
. Spousal abuse
. Bleeding disorders
. Collagen vascular disorders

¡X¡X¡X¡X¡X

Case 30 sim.pt. instruction

If the doctor asks you about anything other than these , just say ¡§ no ,¡¨ or provide an answer that a normal patient might give.

You are a 32-year-old woman who is brought to the emergency department by your husband due to bruises

History of present illness

. Bruises on the right are between the shoulder and elbow
. When asked how you sustained the injury , say , ¡§ My husband told me that I fell down the stairs¡¨
. If the examinee asks for further clarification, say that you have been hit by your husband
. Husband hits you whenever he has a ¡§ rage episode ¡§ - usually once a week
. He dose not hit your children , although they are afraid to go near him wen he has a rage episode
. Husband is an alcoholic , and he almost always has a bottle of bourbon by his side
. Both of your parents live in the same town as you do but they are not aware of the abuse
. You feel that your husband loves you; you love your husband , but are always on edge when he is around and you do not feel safe
. There have been 2 episodes when you thought height kill you (there is a shotgun in the house and you are afraid he might use it)
. You feel that it would be very difficult for you to leave him
. You have never reported the matter to any government of social agency and of not with to do so
. You have a satisfying sexual relationship with him , and you are monogamous
. If the examinee explains that you need not endure such a relationship in which you are always in mortal fear, say that you will think about reporting it to the social welfare agencies and ask for an emergency contact number for the emergency department

Past medical / family / social history

. No prior medical problems
. No medications
. No drug allergies
. Mother and father are healthy
. Married 7 years , live with spouse
. 2 children , bout age 6 and girl age 5
. Tobacco: No
. Alcohol : No
. Recreational drugs : No

Physical examination

Multiple bruises at right upper arm in various stages f healing . Examination is otherwise normal.

¡X¡X¡X¡X¡X

Case 30 sim. pt. checklist

Following the encounter , check which of the following items were performed by the examinee

History of present illness/ review of systems

. Asked how the injuries occurred
. Asked an open-ended question regarding the abuse
. Asked this happens regularly
. Asked how you feel about your husband
. Asked how your husband feels about you
. Asked if you feel safe at home
. Asked if there are nay weapons at home
. Asked about your sexual relationship with your husband
. Asked if you had emergency plan to leave the house if the need were to arise
. Asked bout any other injuries that you had
. Asked if your daily is aware that you are being abused
. Asked about your husband¡¦s alcoholism
. Asked about child abuse at home

Past medical /family/social history

. Asked about past medical issues , hospitalizations , and surgeries
. Asked about current medications
. Asked about allergies
. Asked about tobacco , alcohol , and recreational drug use
. Asked about sexual history
. Asked about occupation
. Asked about personal supports (eg, friends, family)

Examination

. Washed heads before examination
. Examined without gown , not through gown
. Examined shoulder and elbow on affected side
. Check for the injuries

Counseling

. Explained physical findings
. Explained further workup(eg,x-ray)
. Discussed the need for an emergency action plan
. Discussed finding additional support groups in the community
. Gave you emergency contact number and offered ongoing support

Communication skills and professional conduct

. Knocked before entering the room
. Introduced self and greeted you warmly
. Used your name to address you
. Paid attention to what you said and maintained good eye contact
. Asked opened questions
. Asked non-leading questions . Asked one question at a time
. If husband is present , discussed the need to ask additional questions privately
. Did not pressure you to leave your husband , report abuse to authorities , or take additional actions you did not want to take
. Listened to what you said without interrupting me
. Used plain English rather than technical jargon
. Used appropriate transition sentences
. Used appropriate draping techniques
. Expressed empathy and gave appropriate reassurances
. Asked whether you have any concerns/questions

Differential diagnosis

. Spousal abuse

Diagnostic study/studies

. X-ray in involved area(s)

¡X¡X¡X¡X¡X

Case 30 clinical summary

Clinical Skills Evaluation
Case 30 Patient Note

The following represents a typical note for this patient encounter . the detail may vary depending on the information given by the simulated patient.

History : Describe the history you just obtained form this patient . Include only information (Pertinent positives and negatives)  relevant to this patient¡¦s problem(s).

. 32-yo woman with bruises on the right upper extremity
. caused by multiple incidence of altercation/ abuse by her husband
. Husband has frequent ¡§ rage episodes : associated with alcohol abuse
. Patient has not reported abuse to civil authorities of family member

ROS : Negative
PMHx : Noncontributory
PSHx : None
Meds : None
Allergies : None
FHx : Mother and father are healthy
SHx : Married 7 years , lives with spouse and 2 children ; no tobacco , alcohol , or illicit drug use

Physical examinations : Describe any positive and  negative findings relevant  to this patient¡¦s problem(s) . be careful to include only those parts of the examination performed in this encounter.

. Vital signs : Temperature , 37.4¡¦C (99.3F) ; blood pressure ,120/80 mmHg; pulse , 90/min; and respirations , 16/min
. HEENT : PERRLA , EOMI , normal ENT examination , no head trauma
. Neck : No visible injuries
. Musculoskeletal ; Multiple bruises in various stages of hearing on right upper arm
. Neurologic L CN II-XII grossly intact , UE and LE motor strength and reflexes normla and symmetric
. Psychologic : Awake and alert , affected apprehensive but with appropriate range , clear speech

Data interpretation : Based on what you have learned from the history and physical examination, list up to 3 diagnosis that might explain this patient¡¦s complaint(s) . List your diagnoses from most to least likely . For some cases , fewer than 3 diagnosis will be appropriate . the , enter the positive and negative findings form the history and the physical examination (if present) that support each diagnosis . Finally , list initial diagnostic (if any) you would order for each listed diagnosis (eg,restricted physical examination maneuvers, laboratory tests , imaging ECG , etc).

Diagnosis #1 : Intimate partner abuse

History finding(s)
. Recurrent spouse assault
. Spouse with history of alcohol abuse

Physical examination finding(s)
. Multiple bruises in various stages of healing

Diagnostic studies

. x-rays of shoulder, humerus , and elbow
. CBC
. PT/PTT/INR







µoªí¤å³¹®É¶¡2018/09/13 07:55am¡@IP: ¤w³]©w«O±K[¥»¤å¦@ 7712 ¦ì¤¸²Õ]¡@ 
JuanFe 

 

¸ê®Æ: ¦¹·|­û¥Ø«e¤£¦b½u¤W Male ¨°Às ¥Õ¦Ï®y
«Â±æ: 0
¾y¤O: ¾y¤O: 78957
¸gÅç: ¸gÅç: 35986
¨Ó¦Û: ¦t©z¤¤¡@blank
µo¤å: 1118 ½g
ºëµØ: 0 ½g
¦b½u: 47¤Ñ19®É22¤À57¬í
µù¥U: 2013/06/17
Message ¬d¬Ý¡@·j´M¡@³q°T¿ý¡@½Æ»s¡@¤Þ¥Î¡@¦^ÂФ峹¦^ÂС@[²Ä 32 ¼Ó]
 31 Case 31 scenario ( burning during urination )

Doorway information about patient

The patient is a 20-year-old woman who comes to the office due to burning during urination

Vital signs
. Temperature : 38.3¡¦C (100.9F)
. Blood pressure : 110/80 mmHg
. Pulse : 82/min
. Respirations : 16/min

Basic differential diagnosis

. Cystitis
. Pyelonephritis
. Urethritis
. Vulvovaginitis
. Pelvic inflammatory disease

¡X¡X¡X¡X¡X

Case 31 sim.pt. instruction

If the doctor asks you about anything other than these , just say ¡§ no ,¡¨ or provide an answer that a normal patient might give.

You are a 20-year-old woman who has burning with urination

History of present illness

. 4 day of burning with urination
. Fever to 38.3¡¦C (101F) , chills , and rigors
. Urinating 10-12 times a day , sometimes with increased urgency and little urine
. Dull , intermittent pain in the lower pelvic area , greenish vaginal discharge , and occasional blood in the urine
. Similar episode 1 year ago; diagnosed as chlamydia and treated with doxycycline
. Last menstrual period was 2 weeks ago
. New sexual partner for the past 2 months
. You do not use condoms ad have no pain during intercourse

Review of systems

. No back pain
. No nausea , vomiting , diarrhea , or constipation
. No abnormal vaginal bleeding

Past medical / family / social history

. No otters significant past medical issue or surgeries
. Medications ; Oral contraceptive pill
. No drug allergies
. Father and mother are healthy ; no siblings
. Single , lives alone
. Occupation: college student
. Smoking : No
. Alcohol : Occasional heavy drink at parties
. Recreational drugs : no

Physical examination

Abdomen:
. Mild suprapubic discomfort with deep palpation
. Non-distended , normative bowel sounds
. No CVA tenderness

¡X¡X¡X¡X¡X

Case 31 sim. pt. checklist

Following the encounter , check which of the following items were performed by the examinee

History of present illness/ review of systems

. Asked about the onset of dysuria
. Asked about the frequency and urgency
. Asked about hematuria
. Asked about suprapubic , abdominal , and back pain
. Asked about fever and chills
. Asked about nausea and vomiting
. Asked about vaginal discharge and abdominal vaginal bleeding
. Asked bout pain during intercourse
. Asked about last menstrual period
. Asked bout sexual practices and contraceptive methods

Past medical /family/social history

. Asked about similar episodes in the past
. Asked about past medical issues , hospitalizations , and surgeries
. Asked about current medications
. Asked about medication allergies
. Asked about family health
. Asked about tobacco , alcohol , and recreational drug use
. Asked about sexual history
. Asked about occupation

Examination

. Washed heads before examination
. Examined without gown , not through gown
. Auscultated abdomen
. Palpated abdomen (superficial and deep), including suprapubic area
. Palpated / percussed back for constoverbral angle tenderness

Counseling

. Explained physical findings and problems diagnosis
. Explained further workup
. Explained need for pelvic examination

Communication skills and professional conduct

. Knocked before entering the room
. Introduced self and greeted you warmly
. Used your name to address you
. Paid attention to what you said and maintained good eye contact
. Asked opened questions
. Asked non-leading questions . Asked one question at a time
. Listened to what you said without interrupting me
. Used plain English rather than technical jargon
. Used appropriate transition sentences
. Used appropriate draping techniques
. Summarized the history and explained physical findings
. Expressed empathy and gave appropriate reassurances
. Asked whether you have any concerns/questions

Differential diagnosis

. Cystitis
. Pyelonephritis
. Urethritis
. Vulvovaginitis
. Pelvic inflammatory disease

Diagnostic study/studies

. Pelvic examination
. CBC with differential
. Urinalysis
. Culture of urine
. Urine PCR assay for gonorrhea and chlamydia

¡X¡X¡X¡X¡X

Case 31 clinical summary

Clinical Skills Evaluation
Case 31 Patient Note

The following represents a typical note for this patient encounter . the detail may vary depending on the information given by the simulated patient.

History : Describe the history you just obtained form this patient . Include only information (Pertinent positives and negatives)  relevant to this patient¡¦s problem(s).

. 20-yo woman with 4 days of dysuria , increased urinary frequency and urgency , fever , chills , and an episode of hematuria
. New sexual partner 2 months ago with no condom use
. Treated for chlamydia cervicitis a year ago with similar symptoms
. Intermittent suprapubic pain with green vaginal diachange

ROS : No back pain , nausea , vaginal bleeding , pain with intercourse , vomiting , diarrhea , or constipation
PMHx : None
PSHx : None
Meds : Birth control pills
Allergies : None
FHx : Father and mother are healthy
SHx : No smoking , occasion alcohol use

Physical examinations : Describe any positive and  negative findings relevant  to this patient¡¦s problem(s) . be careful to include only those parts of the examination performed in this encounter.

. Vital signs : Temperature, 38.3¡¦C (100.9F) ; blood pressure , 110/80 mmHg ; pulse , 82/min; and respirations , 16/min
. Abdomen : Mild suprapubic discomfort on deep palpation , non-distended , normative bowel sounds , no CVA tenderness

Data interpretation : Based on what you have learned from the history and physical examination, list up to 3 diagnosis that might explain this patient¡¦s complaint(s) . List your diagnoses from most to least likely . For some cases , fewer than 3 diagnosis will be appropriate . the , enter the positive and negative findings form the history and the physical examination (if present) that support each diagnosis . Finally , list initial diagnostic (if any) you would order for each listed diagnosis (eg,restricted physical examination maneuvers, laboratory tests , imaging ECG , etc).

Diagnosis #1 : Cystitis

History finding(s)
. Increased urinary frequency and urgency
. Fever , chills
. Hematuria, dysuria

Physical examination finding(s)
. Suprapubic discomfort
. Fever

Diagnosis #2 : Pyelonephritis

History finding(s)
. Increased urinary frequency and urgency
. Fever , chills
. Hematuria, dysuria

Physical examination finding(s)
. Fever

Diagnosis #3 : Cervicitis

History finding(s)
. Fever
. Vaginal discharge
. New sexual partner with no condom use
. History of previous cervicitis

Physical examination finding(s)
. Fever

Diagnostic studies

. Pelvic examination
. Nucleic acid amplification test for chlamydia and gonorrhea
. Urinalysis
. Urine culture
. CBC with differential








µoªí¤å³¹®É¶¡2018/09/13 07:56am¡@IP: ¤w³]©w«O±K[¥»¤å¦@ 7141 ¦ì¤¸²Õ]¡@ 
JuanFe 

 

¸ê®Æ: ¦¹·|­û¥Ø«e¤£¦b½u¤W Male ¨°Às ¥Õ¦Ï®y
«Â±æ: 0
¾y¤O: ¾y¤O: 78957
¸gÅç: ¸gÅç: 35986
¨Ó¦Û: ¦t©z¤¤¡@blank
µo¤å: 1118 ½g
ºëµØ: 0 ½g
¦b½u: 47¤Ñ19®É22¤À57¬í
µù¥U: 2013/06/17
Message ¬d¬Ý¡@·j´M¡@³q°T¿ý¡@½Æ»s¡@¤Þ¥Î¡@¦^ÂФ峹¦^ÂС@[²Ä 33 ¼Ó]
 32 Case 32 scenario ( difficulty swallowing )

Doorway information about patient

The patient is a 50-year-old man who comes to the office due to difficulty swallowing

Vital signs
. Temperature : 36.7¡¦C (98.1F)
. Blood pressure ; 130/90 mmHg
. Pulse : 85/min
. Respirations : 16/min

Basic differential diagnosis

. Oropharynx dysphagia
- Neuromuscular (stroke , parkinsonism , multiple sclerosis)
- Mechanical obstruction (Zener diverticulum , thyromegaly)
- Skeletal muscle disorders ( myasthenia gravis , muscle dystrophies ,polymyositis)
- Miscellaneous (medication ,radiation)

. Esophageal dysphagia
- Mechanical obstruction (esophageal carcinoma, benign strictures ,webs and rings [Schazki])
- Abnormal motility (achalasia , scleroderma)
- Gastroesphageal reflux disease
- Miscellaneous (diabetes , alcoholism)

¡X¡X¡X¡X¡X

Case 32 sim.pt. instruction

If the doctor asks you about anything other than these , just say ¡§ no ,¡¨ or provide an answer that a normal patient might give.

You are a 50-year-old man who comes to the clinic due to difficulty swallowing

History of present illness

. Onset 3 months ago . Initially had difficulty swallowing soils (food would get stuck in the middle of the chest before slowly going down)
. Symptoms progressed slowly and now you have had difficulty swallowing liquids for the past 3 weeks
. Food regurgitates into the chest 2-3 hours after eating
. No problem chewing of transferring food out of the mouth into the throat
. 10-lb (4.5kg) weight loss in the past 3 months ; decreased appetite for the past 3 weeks

Review of systems

. No weakness in the arm or legs
. No shortness of breath or chest pain
. No nausea , vomiting , diarrhea , or constipation

Past medical / family / social history

. Gastroseophageal reflux (symptoms 2-3times a week for the past 25 years ; relieved with antacids)
. No surgires
. No other medications
. No drug allergies
. Father, mother , and 2 siblings are healthy
. Married , live with wife
. Occupation : Stockbroker
. Tobacco : 1 pack a day for last 30 years
. Alcohol : Occasional wine

Physical examination

¡X¡X¡X¡X¡X

Case 32 sim. pt. checklist

Following the encounter , check which of the following items were performed by the examinee

History of present illness/ review of systems

. Asked an open-ended question about describing the dysphagia
. Asked about the onset and progression over time
. Asked bout exact location where food gets stuck
. Asked whether the dysphagia is for solid , liquid , or both
. Asked which started first( sold or liquids)
. Asked whether there is any associated pain
. Asked about any aggravating or relieving factors
. Asked about episodes of chocking or regurgitation/ aspiration
. Asked about any nausea and vomiting
. Asked about heartburn / gastroesophageal reflux
. Asked about history of ingestion of corrosive materials
. Asked about appetite and changes in weight

Past medical /family/social history

. Asked about similar episodes in the past
. Asked about past medical issues
. Asked bout medications
. Asked about medication allergies
. Asked about family health
. Asked about occupation
. Asked about tobacco , alcohol , and recreational drug use

Examination

. Washed heads before examination
. Examined without gown , not through gown
. Palpated neck for swelling
. Examined mouth and throat
. Gave you water and asked you to swallow
. Palpated lymph nodes in neck , axilla , and about the clavicles
. Auscultated abdomen
. Palpated abdomen(superficial and deep)
. Examined heart and lungs

Counseling

. Explained physical findings and possible diagnosis
. Explained further workup
. Discussed smoking cessation

Communication skills and professional conduct

. Knocked before entering the room
. Introduced self and greeted you warmly
. Used your name to address you
. Paid attention to what you said and maintained good eye contact
. Asked opened questions
. Asked non-leading questions . Asked one question at a time
. Listened to what you said without interrupting me
. Used plain English rather than technical jargon
. Used appropriate transition sentences
. Used appropriate draping techniques
. Summarized the history and explained physical findings
. Expressed empathy and gave appropriate reassurances
. Asked whether you have any concerns/questions

Differential diagnosis

. Carcinoma of the esophagus
. Achalasia
. Reflux esophagitis
. Stricture

Diagnostic study/studies

. CBC
. Esophagram
. Esophagogastroduodenoscopy
. Chest x-ray

¡X¡X¡X¡X¡X

Case 32 clinical summary

Clinical Skills Evaluation
Case 32 Patient Note

The following represents a typical note for this patient encounter . the detail may vary depending on the information given by the simulated patient.

History : Describe the history you just obtained form this patient . Include only information (Pertinent positives and negatives)  relevant to this patient¡¦s problem(s).

. 50-yo man with 3 months of dysphagia , initially with solids and now with liquids for the past 3 weeks.
. No problem with chewing and transferring food to throat , but feels food getting struck in the middle of the chest
. Decreased appetite and 4.-kg (10-lb) weight loss
. Food regurgitation 2-3 hours after eating

ROS : No weakness in the arms or legs , shortness of  breath , nausea , vomiting , chest pain ,diarrhea , constipation , or urinary problems
PMHx : GERD for 25 years ,relieved with OTC antacids
PSHx : None
Meds : OTC antacids
Allergies : None
FHx : Father , mother ,  and 2 sibling are healthy
SHx ; 1 PPD smoker for 30 years , occasional alcohol use

Physical examinations : Describe any positive and  negative findings relevant  to this patient¡¦s problem(s) . be careful to include only those parts of the examination performed in this encounter.

. Vital signs : 36.7¡¦C(98.1F) ; blood pressure , 130/90 mmHg; pulse, 80/min; respirations , 16/min
. HEENT : Oropharynx clear , difficulty swallowing water
. Neck : supple with no lymphadenopathy
. Lymph nodes: No axillary or supraclavicular adenopathy
. Lungs : Clear to auscultation bilaterally
. Heart : RRR with no murmurs , gallops, or rubs
. Abdomen : Non-tender , non-distended , normative bowel sounds , no hepatopslenomeagly, no CVA tenderness

Data interpretation : Based on what you have learned from the history and physical examination, list up to 3 diagnosis that might explain this patient¡¦s complaint(s) . List your diagnoses from most to least likely . For some cases , fewer than 3 diagnosis will be appropriate . the , enter the positive and negative findings form the history and the physical examination (if present) that support each diagnosis . Finally , list initial diagnostic (if any) you would order for each listed diagnosis (eg,restricted physical examination maneuvers, laboratory tests , imaging ECG , etc).

Diagnosis #1 : Esophageal cancer

History finding(s)
. Dysphagia with solids and then liquids
. Weight loss with decreased appetite
. Smoking history

Physical examination finding(s)
. None

Diagnosis #2 : Achalasia

History finding(s)
. Dysphagia with solids and liquids
. Weight loss

Physical examination finding(s)
. None
Diagnosis #3 : Reflux esophagitis / stricture

History finding(s)
. History of GERD
. Food regurgitation 2-3 hours after eating
. OTC antacid use

Physical examination finding(s)
. None

Diagnostic studies

. Chest-x-ray
. Barium swallow
. Upper GI endoscopy







µoªí¤å³¹®É¶¡2018/09/13 07:57am¡@IP: ¤w³]©w«O±K[¥»¤å¦@ 7833 ¦ì¤¸²Õ]¡@ 
JuanFe 

 

¸ê®Æ: ¦¹·|­û¥Ø«e¤£¦b½u¤W Male ¨°Às ¥Õ¦Ï®y
«Â±æ: 0
¾y¤O: ¾y¤O: 78957
¸gÅç: ¸gÅç: 35986
¨Ó¦Û: ¦t©z¤¤¡@blank
µo¤å: 1118 ½g
ºëµØ: 0 ½g
¦b½u: 47¤Ñ19®É22¤À57¬í
µù¥U: 2013/06/17
Message ¬d¬Ý¡@·j´M¡@³q°T¿ý¡@½Æ»s¡@¤Þ¥Î¡@¦^ÂФ峹¦^ÂС@[²Ä 34 ¼Ó]
 33 Case 33 scenario ( refill medication for HIV )

Doorway information about patient

The patient is a 30-year0-old man who comes to the office to refill medications for HIV

Vital signs

. Temperature : 37.1¡¦C (98.8F)
. Blodpressure : 120/75 mmHg
. Pulse : 78/min
. Respirations : 16/min

Basic differential diagnosis

. HIV

¡X¡X¡X¡X¡X

Case 33 sim.pt. instruction

If the doctor asks you about anything other than these , just say ¡§ no ,¡¨ or provide an answer that a normal patient might give.

You are a 30-year-old man requesting a refill for your HIV medications

History of present illness

. Diagnosed with HIV 1 year ago
. Your partner at the time terminated the relationship , and you were sad initially but have slowly recovered and currently have a positive outlook
. You have been compliant with your medications for the past 6 months
. Your CD4 count 3 months ago was 480 with an undetectable viral load

Do not volunteer this information unless asked :

Review of systems

. Normal appetite with no recent change in weight
. No fever , chills , or night sweats
. No  weakness , numbness , or tingling in the extremities
. No chest pain , cough , or shortness of breath
. No abdominal pain , diarrhea , or constipation
. No genital lesion , urethral discharge , or ruining with urination
. No skin lesion or rashes

Past medical / family / social history

. HIV decided on screening 1 year ago
. No prior medical problems
. Medications : HARRT medications
. Allergies : None
. Surgeries : None
. Immediate family members (after , mother , sister ) are all healthy
. Occupation : Truck driver
. Single ; male partners in the past but none in the last year
. Tobacco : No
. Alcohol : No
. Recreational drugs : No

Physical examination

HEENT :
. Oropharynx clear
. Fund without papilledema or lesion

Neck :
. Supple without lymphadenopathy

Lungs :
. Clear to auscultation bilaterally

Heart :
. Regular rate and rhythm
. No murmurs , gallops , or rubs

Abdomen :
. Non-tender , non-distended
. Normative bowel sounds . No hepatosplenomegaly
. No CVA tenderness

Extremities :
. No edema
. No skin rashes

¡X¡X¡X¡X¡X

Case 33 sim. pt. checklist

Following the encounter , check which of the following items were performed by the examinee

History of present illness/ review of systems

. Asked about current medication regimen and compliance
. Asked about side effects of drugs
. Asked about symptoms of possible HIV-related illness
- Systemic symptoms (eg , fever , changes in weight )
- Breathing problems (eg, cough , shortness of breath)
- Headaches - Eye problems (eg , pain , redness , blurred vision)
- Oral ulcers or white patches
- Pain of difficulty with swallowing
- Skin lesion or rashes
- Weakness and sensory symptoms
- Abdominal /bowel problems (eg , pain , cause , vomiting , diarrhea)
- Urogenital problems (dysuria, lesion)
. Asked about  symptoms of depression

Past medical /family/social history

. Asked about past medical issues
. Asked about concurrent medications
. Asked about medical allergies
. Asked about past hospitalizations and surgeries
. Asked about family health
. Asked about tobacco , alcohol , and recreational drug use
. Asked about sexual history
. Asked bout occupation

Examination

. Washed heads before examination
. Examined without gown , not through gown
. Examined eyes, ears, nose , and throat
. Examined lymph nodes
. Examined lungs and heart
. Examined sensation in hands and legs
. Examined abdomen

Counseling

. Explained physical findings and possible additional diagnosis(if any)
. Explained further workup
. Discussed safe sexual practices and use of condoms
. Discussed potential complications and how to deal with them
. Discussed recommended vaccinations

Communication skills and professional conduct

. Knocked before entering the room
. Introduced self and greeted you warmly
. Used your name to address you
. Paid attention to what you said and maintained good eye contact
. Asked opened questions
. Asked non-leading questions . Asked one question at a time
. Listened to what you said without interrupting me
. Used plain English rather than technical jargon
. Used appropriate transition sentences
. Used appropriate draping techniques
. Summarized the history and explained physical findings
. Expressed empathy and gave appropriate reassurances
. Asked whether you have any concerns/questions

Differential diagnosis

. HIV
.(if the simulated patient is instructed to report additional symptoms or signs , consider also: Pneumocytis infection , Candida infection, cytomegalovirus retinitis , esophagitis)

Diagnostic study/studies

. CBC with differential count
. Serum chemistry (including hepatic function markers)
. CD4 cell count
. Viral load (HIV , RNA , PCR)
. Chest x-ray

¡X¡X¡X¡X¡X

Case 33 clinical summary

Clinical Skills Evaluation
Case 33 Patient Note

The following represents a typical note for this patient encounter . the detail may vary depending on the information given by the simulated patient.

History : Describe the history you just obtained form this patient . Include only information (Pertinent positives and negatives)  relevant to this patient¡¦s problem(s).

. 30-yo man needing refill of HIV medications
. Diagnosed a year ago
. Initially felt sad but now has positive outlook
. Compliant with medications
. No weight loss , normal appetite

ROS : No numbness or tingling in the extremities , weakness , chest pain , shortness of breath , abdominal pain . rashes , cough , diarrhea , constipation , genital lesions , fever, chills , or night sweats PMHx : HIV diagnosis a year ago ; 3 months ago , CD4 count was 480/mm3 with undectetable viral load
PSHx : None
Meds : HAART therapy
Allergis : None
FHx : Father , mother , and sister are healthy
SHx : No tobacco or alcohol use

Physical examinations : Describe any positive and  negative findings relevant  to this patient¡¦s problem(s) . be careful to include only those parts of the examination performed in this encounter.

. Vital signs ; Temperature , 37.1¡¦C (98.8F) ; blood pressure , 120/75mmHg; pulse , 78/min; and respirations, 16/min
. HEENT : Oropharynx clear , fund without papilledema or lesions
. Necks ; Supple without lymphadenopathy
. Lungs ; Clear to auscultation bilaterally
. Heart : Regular rate and rhythm without murmurs , gallops, or rubs
. Abdomen ; Non-tender, non-distended , normative bowel sounds , no hepatosplenomegaly, no CVA tenderness
. Extremities ; No rash or edema

Data interpretation : Based on what you have learned from the history and physical examination, list up to 3 diagnosis that might explain this patient¡¦s complaint(s) . List your diagnoses from most to least likely . For some cases , fewer than 3 diagnosis will be appropriate . the , enter the positive and negative findings form the history and the physical examination (if present) that support each diagnosis . Finally , list initial diagnostic (if any) you would order for each listed diagnosis (eg,restricted physical examination maneuvers, laboratory tests , imaging ECG , etc).

Diagnosis #1 : Chronic HIV

History finding(s)
. HIV diagnosis a year ago
. Recent stable CD4 count and viral load
. Complaints with medications

Physical examination finding(s)
. Normal examination findings

Diagnostic studies

. CD4 count and viral load
. CBC with differential
. Liver function tests







µoªí¤å³¹®É¶¡2018/09/13 07:57am¡@IP: ¤w³]©w«O±K[¥»¤å¦@ 7805 ¦ì¤¸²Õ]¡@ 
JuanFe 

 

¸ê®Æ: ¦¹·|­û¥Ø«e¤£¦b½u¤W Male ¨°Às ¥Õ¦Ï®y
«Â±æ: 0
¾y¤O: ¾y¤O: 78957
¸gÅç: ¸gÅç: 35986
¨Ó¦Û: ¦t©z¤¤¡@blank
µo¤å: 1118 ½g
ºëµØ: 0 ½g
¦b½u: 47¤Ñ19®É22¤À57¬í
µù¥U: 2013/06/17
Message ¬d¬Ý¡@·j´M¡@³q°T¿ý¡@½Æ»s¡@¤Þ¥Î¡@¦^ÂФ峹¦^ÂС@[²Ä 35 ¼Ó]
 34 Case 34 scenario ( amenorrhea )

Doorway information about patient

The patient is a 16-year-old girl who comes to the office due to amenorrhea

Vital signs
. Temperature : 36.7¡¦C(98.1F)
. Blood pressure ; 120/70 mmHg
. Pulse : 76/min
. Respirations : 16/min

Basic differential diagnosis

. Pregnancy
. Primary amenorrhea
- Chromosomal disorders
- Abnormal mullerian development
- Androgen insensitive

. Secondary amenorrhea
- Eating disorder
- Hyperprolactinemia
- Thyroid disfunction
- Polycystic ovarian syndorme
- Functional hypothalamic amenorrhea
- Postpill amenorrhea
- Hypothalamic / pituitary mass

¡X¡X¡X¡X¡X

Case 34 sim.pt. instruction

If the doctor asks you about anything other than these , just say ¡§ no ,¡¨ or provide an answer that a normal patient might give.

You are a 16-year-old girl who has not had a menstrual period for 2 months

History of present illness

. Menses began at age 13 and were regular with no missed periods until 2 months ago
. The cycle usually last 28-30 days with moderate bleeding for 3-4 days (3-4 full soaked pads a day)
. Increased stress at school for the past 4 months , and you are currently studying for mid-term examinations in 2 weeks
. 10-lb (4.5kg) weight loss over the last 4 months despite normal appetite

Do not volunteer this information unless asked :

. No palpations
. No diarrhea or constipation
. No hair or skin changes
. No breast tenderness or nipple discharge
. No vaginal discharge or otters genitourinary symptoms

During the interview , ask the examinee : ¡§Do you think I¡¦m pregnant ? I can¡¦t be pregnant right now . My parents will not be happy .¡¨

Past medical / family / social history

. No history of pregnancy (G0P0)
. No surgeries
. No medications
. No drug allergies
. Immediate family members ( father , mother , sister ) are healthy
. Single , live with parents
. Junior in high school ; during well in school and participate in multiple extracurricular activities
. Sexually active with boyfriend for last 6 months ; do not always use a condom
. Tobacco : No
. Alcohol : No
. Recreational drugs : No

Physical examination

HEENT :
. PERRLA , EOMI
. Oropharynx clear

Neck :
. Supple without lymphadenopathy or thyromegaly

Abdomen :
. Non-tender , non-distended
. Normative bowel sounds
. No hepatosplenomegaly

Psychiatric :
. Alert and oriented
. Anxious affect

¡X¡X¡X¡X¡X

Case 34 sim. pt. checklist

Following the encounter , check which of the following items were performed by the examinee

History of present illness/ review of systems

. Asked about the onset and duration of amenorrhea (i.e., last menstrual period)
. Asked about menarche and previous menses (frequency , duration , quantify of blood loss)
. Asked bout associated symptoms:
- Abdominal pain
- Vaginal discharge
- Change I¡¦m appetite or weight
- Cold or heat intolerance
- Changes in skin or hair
- Breast changes or nipple discharges
- Headache
. Asked about sexual activity (and use of contraception )
. Asked about life stressors

Past medical /family/social history

. Asked about similar episodes in the past
. Asked about other medical problems
. Asked about surgeries and gynecologic procedures (eg , dilation and curretage )
. Asked about current and recent medications
. Asked about medication allergies
. Asked about family  health
. Asked about tobacco , alcohol , and recreational drug use

Examination

. Washed heads before examination
. Examined without gown , not through gown
. Examined eyes (including visual fields)
. Examined neck ( including thyroid )
. Examined abdomen

Counseling

. Explained physical findings and possible diagnosis
. Explained further workup
. Discussed the need for pelvic and breadth examinations

Communication skills and professional conduct

. Knocked before entering the room
. Introduced self and greeted you warmly
. Used your name to address you
. Paid attention to what you said and maintained good eye contact
. Asked opened questions
. Asked non-leading questions . Asked one question at a time
. Listened to what you said without interrupting me
. Used plain English rather than technical jargon
. Used appropriate transition sentences
. Used appropriate draping techniques
. Summarized the history and explained physical findings
. Expressed empathy and gave appropriate reassurances
. Asked whether you have any concerns/questions

Differential diagnosis

. Pregnancy
. Eating disorder
. Thyroid dysfunction
. hyperprolactinemia
. Functional hypothalamic amenorrhea

Diagnostic study/studies

. Pelvic and breast examination
. Pregnancy test
. TSH
. Serum prolactin level
. Pelvic ultrasound
. Brain MRI
. LH and FSH levels

¡X¡X¡X¡X¡X

Case 34 clinical summary

Clinical Skills Evaluation
Case 34 Patient Note

The following represents a typical note for this patient encounter . the detail may vary depending on the information given by the simulated patient.

History : Describe the history you just obtained form this patient . Include only information (Pertinent positives and negatives)  relevant to this patient¡¦s problem(s).

. 16-yo girl with 2 month of amenorrhea
. Menarche at age 13 with regular and no missed periods until 2 months ago.
. Sexually active with boyfriend and dose not always use condoms.
. Increased stress at school
. Unintentional 4.52-kg (10-lb ) weight loss in past 4 months with good appetite.

ROS : No palpation , diarrhea , constipation , hair loss  skin changes , breath tenderness , nipple discharge , vaginal discharge , or urinary problems
PMHx : G0P0
PSH : None
Meds : None
Allergies : None
FHx : Father , mother and sister are healthy
SHx : No tobacco or alcohol use

Physical examinations : Describe any positive and  negative findings relevant  to this patient¡¦s problem(s) . be careful to include only those parts of the examination performed in this encounter.

. Vital signs : Temperature ,36.7¡¦C (98.1F); blood pressure 120/70 mmHg; pulse : 76/min; and respiration , 16/min
. HEENT : PERRLA , EOMI , oropharynx clear
. Neck : Supple without lymphadenopathy or thyromegaly
. Abdomen ; Non-tender, non-distended , normative bowel sounds ,. no hepatosplenomegaly Data interpretation : Based on what you have learned from the history and physical examination, list up to 3 diagnosis that might explain this patient¡¦s complaint(s) . List your diagnoses from most to least likely . For some cases , fewer than 3 diagnosis will be appropriate . the , enter the positive and negative findings form the history and the physical examination (if present) that support each diagnosis . Finally , list initial diagnostic (if any) you would order for each listed diagnosis (eg,restricted physical examination maneuvers, laboratory tests , imaging ECG , etc).

Diagnosis #1 : Pregnancy

History finding(s)
. Amenorrhea
. Sexually active, sometimes not using condoms

Physical examination finding(s)
. None

Diagnosis #2 : Hyperthyroidism

History finding(s)
. Amenorrhea
. Increased anxiety
. Weight loss with good appetite

Physical examination finding(s)
. None

Diagnosis #3 :Hyperprolactinemia

History finding(s)
. Amenorrhea

Physical examination finding(s)
. None

Diagnostic studies
. Pregnancy test
. TSH and T4 . Pelvic and breath examination
. Prolactin level







µoªí¤å³¹®É¶¡2018/09/13 07:58am¡@IP: ¤w³]©w«O±K[¥»¤å¦@ 7705 ¦ì¤¸²Õ]¡@ 
JuanFe 

 

¸ê®Æ: ¦¹·|­û¥Ø«e¤£¦b½u¤W Male ¨°Às ¥Õ¦Ï®y
«Â±æ: 0
¾y¤O: ¾y¤O: 78957
¸gÅç: ¸gÅç: 35986
¨Ó¦Û: ¦t©z¤¤¡@blank
µo¤å: 1118 ½g
ºëµØ: 0 ½g
¦b½u: 47¤Ñ19®É22¤À57¬í
µù¥U: 2013/06/17
Message ¬d¬Ý¡@·j´M¡@³q°T¿ý¡@½Æ»s¡@¤Þ¥Î¡@¦^ÂФ峹¦^ÂС@[²Ä 36 ¼Ó]
 35 Case 35 scenario ( right lumbar lower abdominal pain )

Doorway information about patient

The patient is a 35-year-old woman who comes to the office due to acute right lumbar and lower abdominal pain

Vital signs

. Temperature : 38.3¡¦C (100.9F)
. Blood pressure : 110/70 mmHg
. Pulse : 100/min
. Respirations : 16/min

Basic differential diagnosis

. Renal colic . Ovarian torsion
. Urinary tract infection /pyelonephritis
. Pelvic inflammatory disease
. Mittelschmerz
. Appendicitis
. Threatened abortion
. Ectopic pregnancy
. Dysmenorrhea
. Endometriosis
. Fibroids

¡X¡X¡X¡X¡X

Case 35 sim.pt. instruction

If the doctor asks you about anything other than these , just say ¡§ no ,¡¨ or provide an answer that a normal patient might give.

You are a 35-year-old woman who has low back and abdominal pain

History of present illness

. 1 day of right low back pain that radiates to pelvis and lower abdomen
. Sharp pain ; 7/10 severity
. Progressively worsening and is not affected by positional charges
. Preceded by burning with urination for 2-3 days
. Associated with fever , chills , nausea (without vomiting) ,. and intermittent blood in urine

. At the end of the interview say , ¡§ aim in a lot of pain . please make it stop.¡¨

Review of systems

. Last menstrual period 3 weeks ago
. No vaginal discharge
. No chest pain or shortness of breath
. No diarrhea or consipation

Past medical / family / social history

. 2 pregnancies with uncomplicated vaginal delivery(G2P2)
. Pelvic inflammatory disease 2 year ago
. UTI twice 2 years ago treated with antibiotics
. No surgeries or hospitalization
. Medications : None
. No drug allergies
. Father , mother and sister are healthy
. Occupation : Bank teller
. Single , live with a children
. Sexually active with boyfriend and do not usually use condoms
. Tobacco : 1 pack a day for last 15 years
. Alcohol L Occasional beer or wine
. Recreational drugs : None

Physical examination

Abdomen :
. Diffused abdominal discomfort during the examination but no focal tenderness

. Non-distended
. Normative bowel sounds
. No hepatosplenomegaly
. Mild CVA tenderness on the right
. Negative psoas test

¡X¡X¡X¡X¡X

Case 35 sim. pt. checklist

Following the encounter , check which of the following items were performed by the examinee

History of present illness/ review of systems

. Asked about the onset if pain and course over time
. Asked bout the location and radiation of pain
. Asked bout the quality and intensity of pain
. asked bout nay aggravating or relieving factors
. Asked bout associated symptoms , especially :
- Fever and chills
- Nausea and vomiting
- Constipation or diarrhea
- Urinary symptoms (eg, burning , blood in urine, frequency)
- Vaginal bleeding/ discharge
. Asked about last menstrual period and menstrual cycle
. Asked bout sexual practices and use of contraception

Past medical /family/social history

. Asked about similar episodes in the past
. Asked about past medical issues (especially urinary tract infections , pelvic inflammatory disease , kidney stones)
. Asked about hospitalizations and surgeries
. Asked about current medications . Asked bout medication allergies
. Asked about family health
. Asked about occupation
. Asked about tobacco , alcohol , and recreational drug use

Examination

. Washed heads before examination
. Examined without gown , not through gown
. Auscultated abdomen
. Palpated abdomen (superficial and deep)
. Tested for rebound tenderness and rigidity
. Tested for constovertebral angle tenderness
. Tested for signs of appendicitis (eg , psoas test)

Counseling

. Explained physical findings and possible diagnosis
. Discussed the need for pelvic examination
. Explained further workup

Communication skills and professional conduct

. Knocked before entering the room
. Introduced self and greeted you warmly
. Used your name to address you
. Paid attention to what you said and maintained good eye contact
. Asked opened questions
. Asked non-leading questions . Asked one question at a time
. Listened to what you said without interrupting me
. Used plain English rather than technical jargon
. Used appropriate transition sentences
. Used appropriate draping techniques
. Summarized the history and explained physical findings
. Expressed empathy and gave appropriate reassurances
. Asked whether you have any concerns/questions

Differential diagnosis

. Renal colic
. UTI/ acute pyelonephritis
. Pelvic inflammatory disease
. Uterine fibrosis
. Appendicitis

Diagnostic study/studies

. Pelvic examination
. Pregnancy test
. CBC with differential count
. Urinary and culture
. Abdomen ultrasound
. Urine PCR for gonorrhea and chlamydia

¡X¡X¡X¡X¡X

Case 35 clinical summary

Clinical Skills Evaluation
Case  Patient Note

The following represents a typical note for this patient encounter . the detail may vary depending on the information given by the simulated patient.

History : Describe the history you just obtained form this patient . Include only information (Pertinent positives and negatives)  relevant to this patient¡¦s problem(s).

. 35-yo woman with 1 day of ship right flank pain radiating to the right pelvis and lower abdomen
. Pain is 7 on scale of 10
. Progressive , worsening pain without relief and unaffected by position
. Dysuria , fever , chills , nausea , and occasional hematuria

ROS : No vaginal discharge , chest pain , shortness pf breath , diarrhea , or constipation
PMHx : G2P2 , PID 2 years ago , UTI 2 years ago
PSHx : None Meds : None
Allergies ; None
FHx : Father , mother , and sister are healthy
SHx : 1 PPD smoker for 15 years , occasional alcohol use

Physical examinations : Describe any positive and  negative findings relevant  to this patient¡¦s problem(s) . be careful to include only those parts of the examination performed in this encounter.

. Vital signs : Temperature , 38.3¡¦C (100.9¡¦F) ; blood pressure , 110/70 mmHg; pulse 100/min; and respirations , 16/min
. Abdomen : Diffuse abdominal discomfort without focal tenderness , non-distended , normative bowel sounds , no hepatosplenomegaly , mild CVA tenderness son the right , negative psoas test

Data interpretation : Based on what you have learned from the history and physical examination, list up to 3 diagnosis that might explain this patient¡¦s complaint(s) . List your diagnoses from most to least likely . For some cases , fewer than 3 diagnosis will be appropriate . the , enter the positive and negative findings form the history and the physical examination (if present) that support each diagnosis . Finally , list initial diagnostic (if any) you would order for each listed diagnosis (eg,restricted physical examination maneuvers, laboratory tests , imaging ECG , etc).

Diagnosis #1 : Nephrolithiasius with hydronephrosis

History finding(s)
. Frank pain radiating to lower abdomen
. No comfort with any position change

Physical examination finding(s)
. Fever
. CVA tenderness

Diagnosis #2 : Pyelonephritis

History finding(s)
. Dysuria
. Hematuria
. Fever

Physical examination finding(s)
. CVA tenderness
. Fever

Diagnosis #3 : Pelvic inflammatory disease

History finding(s)
. Previous PID
. Sexually active without condom use
. Lower abdominal pain

Physical examination finding(s)
. None

Diagnostic studies
. Pregnancy test
. Pelvic examination
. CBC with differential
. Urinalysis and urine culture







µoªí¤å³¹®É¶¡2018/09/13 07:59am¡@IP: ¤w³]©w«O±K[¥»¤å¦@ 7822 ¦ì¤¸²Õ]¡@ 
JuanFe 

 

¸ê®Æ: ¦¹·|­û¥Ø«e¤£¦b½u¤W Male ¨°Às ¥Õ¦Ï®y
«Â±æ: 0
¾y¤O: ¾y¤O: 78957
¸gÅç: ¸gÅç: 35986
¨Ó¦Û: ¦t©z¤¤¡@blank
µo¤å: 1118 ½g
ºëµØ: 0 ½g
¦b½u: 47¤Ñ19®É22¤À57¬í
µù¥U: 2013/06/17
Message ¬d¬Ý¡@·j´M¡@³q°T¿ý¡@½Æ»s¡@¤Þ¥Î¡@¦^ÂФ峹¦^ÂС@[²Ä 37 ¼Ó]
 36 Case 36 scenario ( insomnia )

Doorway information about patient

The patient is a 70-year-old man wth insomnia

Vital signs

. Temperature ; 36.7¡¦C(98.1F)
. Blood pressure : 130/90 mmHg
. Pulse ; 58/min
. Respirations : 16/min

Basic differential diagnosis

. Depression
. Post-Traumatic stress disorder
. Anxiety disorder
. Chronic pan syndormes
. Adverse effect of medication
. Age-related sleep change
. Thyroid problems
. Sleep apnea
. Restless legs syndorme

¡X¡X¡X¡X¡X

Case 36 sim.pt. instruction

If the doctor asks you about anything other than these , just say ¡§ no ,¡¨ or provide an answer that a normal patient might give.

You are a 70-year-old man who has insomnia

History of present illness

. Admitted to the hospital 3 months ago with chest pain and diagnosed with coronary artery disease . Coronary angioplasty was performed and several new medications were initiated . you were very anxious throughout the hospitalization and were given lorazepam for anxiety . A few weeks after discharge , you began having difficulty falling asleep and are able to stay asleep for only 2-3 hours before getting up and pacing.
. No unusual dreams or nightmares
. You do not wake refreshed and feel tired in the morning and throughout the day
. You do not drink alcohol or caffeine in the evening before you go to bed
. Decreased appetite an a 2.27-kg (5-lb) weight loss since the hospitalization
. Your son loves nearby and says that you have become more isolated and not interested in normal activities

Review of systems

. No chest pain
. No shortness of breath or swelling in the ankles /feet
. No tremors or change in strength or sensation
. No changes in hair or skin
. No nausea , vomiting , diarrhea , constipation , or abdominal pain
. No palpitations or dizziness

Past medical / family / social history

. Coronary artery disease
. no surgires
. Medications ; Aspiri , clopidogrel , metoprolol, atovastatin , lisinpril , nitroglycerin, sublingual as needed (have not used)
. No drug allergies . After died at age 75 of heart attack , motor died at age 68 of breath cancer , 1 sister (healthy)
. Widowed for last 2 years , live alone
. Retired accountant
. Tobacco : 1 pack a day for last 50 years
. Alcohol : Occasional beer
. Recreational; drugs : No

Physical examination

General :
. Awake and alert but appear fatigued
. Grooming and hygiene normal
. No distress

HEENT :
. Oropharynx clear

Neck :
. Supple without lymphadenopathy

Lungs :
. Clear to auscultation bilaterally

Heart :
. Regular rate and rhythm
. no murmurs , gallops ,or rubs

Abdomen :
. Non-tender, non-distended
. Normative bowel sounds
. No hepatosplenomegaly

Neurologic :
. Oriented to person , place, and time
. Motor 5/5 throughout
. Reflexes 2+ throughout

¡X¡X¡X¡X¡X

Case 36 sim. pt. checklist

Following the encounter , check which of the following items were performed by the examinee

History of present illness/ review of systems

. Asked about the onset of symptoms
. Asked bout the average duration and quality of sleep
. Asked if having difficulty falling asleep ,staying asleep , or both
. Asked about bedtime habits
. Asked if having nightmares
. Asked if having anxiety or depressive symptoms
. Asked if having any associated palpitations , sweating , or dizziness
. Asked if having any pain
. Asked about snoring / breathing problems
. Asked about daytime sleepiness and morning headaches
. Asked about appetite and changes in weight
. Asked about constipation and diarrhea
. Asked bout impact on personal relationship and daily activities

Past medical /family/social history

. Asked about similar episodes in the past
. Asked about past medical issues , hospitalizations ,and surgeries
. Asked about curent medications
. Asked about medication allergies
. Asked about family health
. Asked bout current living situation and family support
. Asked about occupation

Examination

. Washed heads before examination
. Examined without gown , not through gown
. Examined pharynx
. Examined neck/thyroid
. Performed neurologic examination including cranial nerves , motor strength , and reflexes
. Examined heart and lungs

Counseling

. Explained physical findings and possible diagnosis
. Explained further workup
. Discussed sleep habits/ sleep hygiene
. Discussed smoking cessation and readiness to quit

Communication skills and professional conduct

. Knocked before entering the room
. Introduced self and greeted you warmly
. Used your name to address you
. Paid attention to what you said and maintained good eye contact
. Asked opened questions
. Asked non-leading questions . Asked one question at a time
. Listened to what you said without interrupting me
. Used plain English rather than technical jargon
. Used appropriate transition sentences
. Used appropriate draping techniques
. Summarized the history and explained physical findings
. Expressed empathy and gave appropriate reassurances
. Asked whether you have any concerns/questions

Differential diagnosis

. Anxiety disorder
. Drug-induced insomnia
. Depression
. Sleep apnea

Diagnostic study/studies

. CBC
. Basic metabolic panal (Na, K , BUN , Cr, CO2 , Cl)
. TSH
. Nocturnal polysomnography

¡X¡X¡X¡X¡X

Case 36 clinical summary

Clinical Skills Evaluation
Case 36 Patient Note

The following represents a typical note for this patient encounter . the detail may vary depending on the information given by the simulated patient.

History : Describe the history you just obtained form this patient . Include only information (Pertinent positives and negatives)  relevant to this patient¡¦s problem(s).

. 70yo man with 2 months of insomnia after hospitalization for CAD with angioplasty
. Difficulty falling and staying asleep
. Decreased appetite and 2.27-kg (5-lb ) weight loss.
. Family thinks the patient feels isolated and has loss interest in activities

ROS : Fatigue , no chest pain , shortness of breath , tremor , hair loss diarrhea , constipation, palpitations , dizziness , or recent trauma
PMHx : CAD
PSHx : None
Meds : Aspirin , clopidogrel , metoprolol, lisinpril, atovastatin, nitroglycerin as needed
Allergies : None
FHx : Father died of MI ; mother died of breast cancer
SHx: 1 PPD smoker for 50 years , occasional alcohol use

. Vital signs : Temperature ,36.7¡¦C(98.1F); blood pressure , 130/90 mmHg; pulse , 58/min; respirations , 16/min
. HEENT : Oropharynx clear
. Neck : Supple without lymphadenopathy
. Heart : RRR without murmurs , gallops , or rubs
. Abdomen ; Non-tender, non-distended, normative bowel sounds , no hepatosplenomegaly
. Neurologic : Motor 5/5 bilaterally ; alert and oriented to person , place , and time ; DTR 2+ bilaterally

Physical examinations : Describe any positive and  negative findings relevant  to this patient¡¦s problem(s) . be careful to include only those parts of the examination performed in this encounter.

Data interpretation : Based on what you have learned from the history and physical examination, list up to 3 diagnosis that might explain this patient¡¦s complaint(s) . List your diagnoses from most to least likely . For some cases , fewer than 3 diagnosis will be appropriate . the , enter the positive and negative findings form the history and the physical examination (if present) that support each diagnosis . Finally , list initial diagnostic (if any) you would order for each listed diagnosis (eg,restricted physical examination maneuvers, laboratory tests , imaging ECG , etc).

Diagnosis #1 : Anxiety disorder

History finding(s)
. Recent hospitalization
. Difficulty falling and staying asleep
. Previous anxiety requiring medications

Physical examination finding(s)
. None

Diagnosis #2 : Depression

History finding(s)
. Decreased appetite and weight loss
. Decreased interest in activities
. Insomnia , fatigue

Physical examination finding(s)
. None

Diagnosis #3 : Drug induced insomnia

History finding(s)
. Recently started metoprolol
. Insomnia

Physical examination finding(s)
. Bradycardia

Diagnostic studies

. Basic metabolic panel
. CBC with differential
. TSH








µoªí¤å³¹®É¶¡2018/09/13 08:00am¡@IP: ¤w³]©w«O±K[¥»¤å¦@ 8535 ¦ì¤¸²Õ]¡@ 
JuanFe 

 

¸ê®Æ: ¦¹·|­û¥Ø«e¤£¦b½u¤W Male ¨°Às ¥Õ¦Ï®y
«Â±æ: 0
¾y¤O: ¾y¤O: 78957
¸gÅç: ¸gÅç: 35986
¨Ó¦Û: ¦t©z¤¤¡@blank
µo¤å: 1118 ½g
ºëµØ: 0 ½g
¦b½u: 47¤Ñ19®É22¤À57¬í
µù¥U: 2013/06/17
Message ¬d¬Ý¡@·j´M¡@³q°T¿ý¡@½Æ»s¡@¤Þ¥Î¡@¦^ÂФ峹¦^ÂС@[²Ä 38 ¼Ó]
 37 Case 37 scenario ( difficulty urination)

Doorway information about patient

. The patient is a 65-year-old man who comes to the office due to difficulty with urination

Vital signs
. Temperature : 37.2¡¦C(99F)
. Blood pressure ; 130/80 mmHg
. Pulse ; 92/min
. Respirations : 16/min

Basic differential diagnosis

. Benign prostate hyperplasia
. Prostatitis
. UTI / cystitis
. Carcinoma of the prostate
. Stone in the urinary tract (obstructive)
. Carcinoma of the bladder
. Neurologic dysfunction
. Drug-induced bladder dysfunction

¡X¡X¡X¡X¡X

Case 37 sim.pt. instruction

If the doctor asks you about anything other than these , just say ¡§ no ,¡¨ or provide an answer that a normal patient might give.

History of present illness

. Onset of symptoms 2 months ago
. Difficulty initiating urine stream with decreased flow , straining with urination , sensation of incomplete emptying , and increased urinary frequency
. Mild burning on urination and 1 episode of blood in the urine
. Getting up 5-6 times a night to urinate
. Asked doctor :¡¨ Do you think I have prostate cancer?¡¨

Review of systems

. Decreased appetite with 4.5-kg (10-lb) weight loss over the last year
. No fever or chills
. No  abdominal pain , diarrhea , or constipation
. No muscle weakness
. No recent trauma

Past medical / family / social history

. Diabetes mellitus for the past 10 years
. No surgeries or hospitalizations
. Medications : Metformin 500mg twice daily
. No drug allergies
. Father died of prostate cancer at age 75 , mother died of ¡§kidney problems¡¨ at age 78 , sister is healthy
. Occupation : Accountant
. Married , lived with wife
. Tobacco : No
. Alcohol : 2 beers /day for last 35 years
. Recreational drugs : No

Physical examination

Abdomen :
. Non-tender, non-distended
. Normative bowel sounds
. No hepatosplenomegaly
. No CVA or suprapubic tenderness

Neurologic :
. Motor 5/5 throughout
. DTR 2+bilaterally

¡X¡X¡X¡X¡X

Case 37 sim. pt. checklist

Following the encounter , check which of the following items were performed by the examinee

History of present illness/ review of systems

. Asked about the onset of symptoms and course over time
. Asked about difficulty initiating or maintaining urinary flow
. Asked bout the strength of flow
. Asked about intermittency (stopping and starting again while urinating), straining, and sensation of incomplete emptying
. Asked about the frequency or urination
. Asked about urgency
. Asked about nocturia
. Asked bout any burning sensations with urination
. Asked if any blood in the urine
. Asked about associated symptoms , especially :
- Abdominal pain
- Fever
- Weakness in legs
- Change in bowel movements
- Change in appetite or weight
- Back pain or trauma Past medical /family/social history

. Asked about similar episode sin the past
. Asked about past medical issues(especially urinary or sexually transmitted infections), Surgeries , and hospitalizations
. Asked about current medications
. Asked about medication allergies
. Asked about family health
. Asked about tobacco, alcohol , and recreational drug use
. Asked about occupation

Examination

. Washed heads before examination
. Examined without gown , not through gown
. Examined abdomen  (auscultation ,palpation)
. Examined back , including palpation/ percussion for ocstovertebral angle tenderness
. Tested lower extremity strength and reflexes

Counseling

. Explained physical findings and possible diagnosis
. Explained further  workup
. Explained the need for rectal / prostate examination

Communication skills and professional conduct

. Knocked before entering the room
. Introduced self and greeted you warmly
. Used your name to address you
. Paid attention to what you said and maintained good eye contact
. Asked opened questions
. Asked non-leading questions . Asked one question at a time
. Listened to what you said without interrupting me
. Used plain English rather than technical jargon
. Used appropriate transition sentences
. Used appropriate draping techniques
. Summarized the history and explained physical findings
. Expressed empathy and gave appropriate reassurances
. Asked whether you have any concerns/questions

Differential diagnosis

. Benign prostate hyperplasia
. Urinary tract infection
. Prostatitis
. Prostatic carcinoma
. Bladder carcinoma
. Overflow incontinence

Diagnostic study/studies

. Rectal examination
. urinalysis andurine culture
. Serum BUN , Creatinine , glucose
. Hemoglobin A1c
. CBC with differential
. Prostate-specific antigen

¡X¡X¡X¡X¡X

Case 37 clinical summary

Clinical Skills Evaluation
Case 37 Patient Note

The following represents a typical note for this patient encounter . the detail may vary depending on the information given by the simulated patient.

History : Describe the history you just obtained form this patient . Include only information (Pertinent positives and negatives)  relevant to this patient¡¦s problem(s).

. 95-yo man with 2 months of difficulty initiating urination , decreased urinary flow , nocturia , increase urinary frequency , and incomplete emptying of bladder
. 1 episode of hematuria
. Decreased appetite and 4.53kg(10-lb) weight loss over the past year.

ROS : No increased urinary urgency , abdominal pain, flank pain , fever , chills , diarrhea , constipation, leg weakness , or trauma
PMHx : Diarrhea for past 10 years
PSHx : None
Meds : Mptformin 500 mg twice a day
Allergies : None
FHx : Father died of prostate cancer ; motor died of kidney problems
SHx : No tobacco use l 2 beers daily for 35 years

Physical examinations : Describe any positive and  negative findings relevant  to this patient¡¦s problem(s) . be careful to include only those parts of the examination performed in this encounter.

. Vital signs: Temperature ,37.2;C(99F); blood pressure , 130/80 mmHg; pulse, 92/min; and respirations, 16/min
. Abdomen : Non-tender, non-distended , normative bowel sounds , no hepatosplenomegaly, no CVA tenderness
. Neurologic : Motor 5/5 bilaterally , DTR 2+ bilaterally

Data interpretation : Based on what you have learned from the history and physical examination, list up to 3 diagnosis that might explain this patient¡¦s complaint(s) . List your diagnoses from most to least likely . For some cases , fewer than 3 diagnosis will be appropriate . the , enter the positive and negative findings form the history and the physical examination (if present) that support each diagnosis . Finally , list initial diagnostic (if any) you would order for each listed diagnosis (eg,restricted physical examination maneuvers, laboratory tests , imaging ECG , etc).

Diagnosis #1 : Benign prostate hyperplasia

History finding(s)
. Difficulty with urination
. Incomplete emptying of bladder
. Nocturia, decreased urinary flow

Physical examination finding(s)
. None

Diagnosis #2 : Prostate cancer

History finding(s)
. Family history of prostate cancer
.Decreased urinary flow, nocturia
. Weight loss

Physical examination finding(s)
. None

Diagnosis #3 : Bladder cancer

History finding(s)
. Gross hematuria
. Incomplete emptying of bladder
. Weight loss

Physical examination finding(s)
. None

Diagnostic studies

. Rectal examination
. Urinalysis with curse culture
. PSA
. Basic metabolic panel







µoªí¤å³¹®É¶¡2018/09/13 08:01am¡@IP: ¤w³]©w«O±K[¥»¤å¦@ 7696 ¦ì¤¸²Õ]¡@ 
JuanFe 

 

¸ê®Æ: ¦¹·|­û¥Ø«e¤£¦b½u¤W Male ¨°Às ¥Õ¦Ï®y
«Â±æ: 0
¾y¤O: ¾y¤O: 78957
¸gÅç: ¸gÅç: 35986
¨Ó¦Û: ¦t©z¤¤¡@blank
µo¤å: 1118 ½g
ºëµØ: 0 ½g
¦b½u: 47¤Ñ19®É22¤À57¬í
µù¥U: 2013/06/17
Message ¬d¬Ý¡@·j´M¡@³q°T¿ý¡@½Æ»s¡@¤Þ¥Î¡@¦^ÂФ峹¦^ÂС@[²Ä 39 ¼Ó]
 38 Case 38 scenario ( anxiety )

Doorway information about patient

The patient is a 35-year-old woman who comes to the emergency department due to breathlessness and anxiety.

Vital signs

. Temperature : 36.1¡¦C(97F)
. Blood pressure : 130/80 mmHg
. Pulse ; 94/min
. Respirations : 22/min

Basic differential diagnosis

. Anxiety secondary to medical condition (eg, hyperthyroidism, arrhythmias)
. Substance abuse
. Panic disorder
. Generalized anxiety disorder
. Adjustment disorder with anxious mood
. Acute stress disorder or post-traumatic stress disorder
. Hypochondriasis

¡X¡X¡X¡X¡X

Case 38 sim.pt. instruction

If the doctor asks you about anything other than these , just say ¡§ no ,¡¨ or provide an answer that a normal patient might give.

You are a 35-year-old woman experiencing shortness of breath

History of present illness

. Episodic shortness of breath for 3 months ; slight problems previously but never this severe
. Episodes last 30 minutes and are associated with palpitations , sweating ,and feeling that you are going to die
. Episodes occur about 2 or 3 times a week at any time but are worse in crowded places outside the house , and you have stopped going to outdoor activities to avoid triggering symptoms
. Symptoms seem to improve with sloe breathing and relaxation
. Multiple emergency department evaluations for the same symptoms ; all test have been normal/nondiagnostic
. Ask the doctor : ¡§ Do you think that this is anxiety like my mother has?¡¨

Review of systems

. No chest pain
. No headaches or tremors
. Occasional diarrhea alternating with constipation
. No nausea , vomiting , or abdominal pain

Past medical / family / social history

. No prior medical issues , surgeries , or hospitalizations
. No medications
. Allergies : Penicillin causes a rash
. Father is healthy , mother has generalized anxiety disorder, sister is healthy
. Married , live with husband and 2 children
. Occupation : Homemaker
. Tobacco : No
. Alcohol : Wine on social occasions only
. Recreational drugs : Used marijuana occasionally in college but non since then
. Caffeine : 1 cup of coffee daily

Physical examination

Neck:
. Supple without thyromegaly or lymphadenopathy

Lungs :
. Clear to auscultation

Heart :
. Regular rhythm
. No nurtures, rubs, or gallops

Neurologic :
. No treor in extremities

Psychological :
. Alert and oriented
. Affect mildly anxious but otherwise appropriate
. Speech clear

¡X¡X¡X¡X¡X

Case 38 sim. pt. checklist

Following the encounter , check which of the following items were performed by the examinee

History of present illness/ review of systems

. Asked about the initial onset of symptoms and course over time
. Asked about the frequency and duration of attacks
. Asked about associated symptoms . especially :
- Chest pain
- Swelling in neck
- Fear/apprehension, sense of impending doom
- Palpitations
- Dizziness
- Tremor
- Sweating
. Asked about aggravating and relieving factors
. Asked about impact of symptoms on relationship and normal activities

Past medical /family/social history

. Asked about similar episodes in the past
. Asked about past medical issues (especially thyroid and psychological disorders)
. Asked about previous hospitalizations and surgeries
. Asked about current medications
. Asked about medication allergies
. Asked about family health
. Asked about occupation
. Asked bout tobacco , alcohol , and recreational drugs

Examination

. Washed heads before examination
. Examined without gown , not through gown
. Palpated neck for swelling
. Examined hands for tremor
. Examined heart and lungs
. Examined cranial nerves , motor strength , and reflexes

Counseling

. Explained the physical findings and possible diagnosis
. Explained further workup (if any)

Communication skills and professional conduct

. Knocked before entering the room
. Introduced self and greeted you warmly
. Used your name to address you
. Paid attention to what you said and maintained good eye contact
. Asked opened questions
. Asked non-leading questions . Asked one question at a time
. Listened to what you said without interrupting me
. Used plain English rather than technical jargon
. Used appropriate transition sentences
. Used appropriate draping techniques
. Summarized the history and explained physical findings
. Expressed empathy and gave appropriate reassurances
. Asked whether you have any concerns/questions

Differential diagnosis

. Panic disorder/ agoraphobia
. Generalized anxiety disorder
. Hyperthyroidism
. Substance abuse

Diagnostic study/studies

. ECG
. Electrolytes and glucose
. TSH
. Urine drug screen

¡X¡X¡X¡X¡X

Case 38 clinical summary

Clinical Skills Evaluation
Case 38 Patient Note

The following represents a typical note for this patient encounter . the detail may vary depending on the information given by the simulated patient.

History : Describe the history you just obtained form this patient . Include only information (Pertinent positives and negatives)  relevant to this patient¡¦s problem(s).

.35-yo woman with 3 months of episodic shortness of breath , palpitations , diaphoresis, and feeling of impending death
. Episodes lasting 30 minutes and occurring more frequently outside of house in crowded places
. Symptom improvement with slow breathing and relaxation
. Multiple ED trips with normal investigations and no definitive diagnosis

ROS : Occasional diarrhea alternating with constipation ; no chest pain, headache nausea , vomiting , tremors , neck swelling , or abdominal pain
PMHx : None
PSHx : None
Meds : None
Allergies : Penicillin (rash)
FHx : Father is healthy ; mother has generalized anxiety disorder
SHx ; No tobacco use , occasional glass of wine

Physical examinations : Describe any positive and  negative findings relevant  to this patient¡¦s problem(s) . be careful to include only those parts of the examination performed in this encounter.

. Vital signs ; Temperature ,36.1¡¦C(97F); blood pressure , 130/80/mmHg; pulse ,94/min; and respirations , 22/min
. Neck : Supple without thyromegaly or lymphadenopathy
. Lung : Clear to auscultation
. Heart : Regular rhythm without murmurs , rubs, gallops
. Neurologic ; No tremor in extremities

Data interpretation : Based on what you have learned from the history and physical examination, list up to 3 diagnosis that might explain this patient¡¦s complaint(s) . List your diagnoses from most to least likely . For some cases , fewer than 3 diagnosis will be appropriate . the , enter the positive and negative findings form the history and the physical examination (if present) that support each diagnosis . Finally , list initial diagnostic (if any) you would order for each listed diagnosis (eg,restricted physical examination maneuvers, laboratory tests , imaging ECG , etc).

Diagnosis #1 : Panic disorder

History finding(s)
. Episodes of palpitations with dyspnea
. Family history of anxiety
. Symptoms worse in crowded places
. Symptoms relieved wth slow breathing

Physical examination finding(s)
. None

Diagnosis #2 : Hyperthyroidism

History finding(s)
. Episodes of palpitations
. Shortness of breath and diaphoresis

Physical examination finding(s)
. None

Diagnosis #3 : Cardiac arrhythmia

History finding(s)
. Palpitations
. Shortness of breath and diaphoresis

Physical examination finding(s)
. None

Diagnostic studies

. ECG
. TSH
. Serum electrolytes and glucose








µoªí¤å³¹®É¶¡2018/09/13 08:02am¡@IP: ¤w³]©w«O±K[¥»¤å¦@ 7835 ¦ì¤¸²Õ]¡@ 
JuanFe 

 

¸ê®Æ: ¦¹·|­û¥Ø«e¤£¦b½u¤W Male ¨°Às ¥Õ¦Ï®y
«Â±æ: 0
¾y¤O: ¾y¤O: 78957
¸gÅç: ¸gÅç: 35986
¨Ó¦Û: ¦t©z¤¤¡@blank
µo¤å: 1118 ½g
ºëµØ: 0 ½g
¦b½u: 47¤Ñ19®É22¤À57¬í
µù¥U: 2013/06/17
Message ¬d¬Ý¡@·j´M¡@³q°T¿ý¡@½Æ»s¡@¤Þ¥Î¡@¦^ÂФ峹¦^ÂС@[²Ä 40 ¼Ó]
 [³o½g¤å³¹³Ì«á¥ÑJuanFe¦b 2019/05/23 08:10pm ²Ä 1 ¦¸½s¿è]
39 Case 39 scenario

Doorway information about patient ( epigastric pain )

The patient is a 53-year-old man who comes to the emergency department due to epigastric pain

Vital signs

. Temperature : 36.1¡¦C
. Blood pressure : 120/70 mmHg
. Pulse : 84 /min
. Respirations : 16/min

Abdominal x-ray is as shown in the exhibit


¦¹¥DÃD¬ÛÃö¹Ï¤ù¦p¤U¡G
«ö¦¹¦b·sµøµ¡ÂsÄý¹Ï¤ù

«ö¦¹¬d¬Ý¹Ï¤ù¸Ô²Ó¸ê®Æ



¦¹¥DÃD¬ÛÃö¹Ï¤ù¦p¤U¡G
«ö¦¹¦b·sµøµ¡ÂsÄý¹Ï¤ù

«ö¦¹¬d¬Ý¹Ï¤ù¸Ô²Ó¸ê®Æ



Basic differential diagnosis

. Peptic ulcer
. Gastritis
. Esophagitis(GERD)
. Carcinoma of esophagus , stomach , or pancreas
. Acute or chronic pancreatitis
. Cholecystitis
. Hepatitis
. Acute coronary event

¡X¡X¡X¡X¡X

Case 39 sim.pt. instruction

If the doctor asks you about anything other than these , just say ¡§ no ,¡¨ or provide an answer that a normal patient might give.

You are a 53-year-old man who abdominal pain

History of present illness

. Intermittent abdominal pain for the past 2 years
. Located in midepigastric area and sometimes radiates to back
. Sharp quality ; 7/10 severity at worst
. Worse with meals and sometimes relieved with antacids; the pain also occurs in the middle of the night
. Associated symptoms :
- Decreased appetite with 6.8-kg(15-lb) weight loss in the last 6 months
- Abdominal bloating and feeling of fullness
- Occasional black stools . Asked the doctor : ¡¨ Can you please stop this pain ? Is it durable?¡¨ Review of systems

. No fever or chills . No jaundice
. No shortness of breath
. No nausea , vomiting , diarrhea , or constipation

Past medical / family / social history

. Osteoarthritis of the knee for past 10 years
. Surgeries : None
. Medications : Ibuprofen 600 mg 3 times a day as needed , over-the-counter antacids as needed
. No drug allergies
. Father is healthy , mother died of pancreatic cancer at age 60, broth is healthy
. Occupation ; stockbroker
. Married , live with wife and 2 children
. Tobacco : No

Physical examination

Neck :
. Supple without thyromegaly or lymphadenopathy

Abdomen :
. Soft , non-tender, non-disveended
. Normative bowel sounds throughout
. No hepatosplenomegaly
. No bruits

¡X¡X¡X¡X¡X

Case 39 sim. pt. checklist

Following the encounter , check which of the following items were performed by the examinee

History of present illness/ review of systems

. Asked about the onset of pain
. Asked about the course over time
. Asked about the location and radiation of pain
. Asked about the quality and intensity of pain
. Asked about any aggravating or relieving factors (especially with relation of food)
. Asked bout associated symptoms , especially :
- Nausea
- Vomiting
- Heartburn
- Black stools or red blood in stools
- Jaundice
- Changes in appetite or weight
. Asked about dietary and bowel habits
. Asked about postprandial fullness or early satisfy

Past medical /family/social history

. Asked about similar episodes in the past
. Asked about previous medical issues , hospitalizations ,and surgeries
. Asked about current medications
. Asked bout medication allergies
. Asked bout family health
. Asked bout tobacco , alcohol , and recreational drug use
. Asked about occupation

Examination

. Washed heads before examination
. Examined without gown , not through gown
. Examined heart and lungs
. Examined abdomen (auscultation ,  superficial and deep palpation)
. Palpated axilla and above clavicle for lymph nodes

Counseling

. Explained physical findings and possible diagnosis
. Explained further workup

Communication skills and professional conduct

. Knocked before entering the room
. Introduced self and greeted you warmly
. Used your name to address you
. Paid attention to what you said and maintained good eye contact
. Asked opened questions
. Asked non-leading questions . Asked one question at a time
. Listened to what you said without interrupting me
. Used plain English rather than technical jargon
. Used appropriate transition sentences
. Used appropriate draping techniques
. Summarized the history and explained physical findings
. Expressed empathy and gave appropriate reassurances
. Asked whether you have any concerns/questions

Differential diagnosis

. Gastritis (NSAID gastropathy)
. Peptic ulcer
. GERD
. Gastric or pancreatic carcinoma
. Chronic pancreatitis

Diagnostic study/studies

. CBC with differential count
. Upper GI endoscopy
. Serum amylase and lipase
. Liver  function tests (albumin , bilirubin, AST , ALT , alkaline phosphatase)
. Fecal occult blood test
. Abdomen ultrasound or CT scan

¡X¡X¡X¡X¡X

Case 39 clinical summary

Clinical Skills Evaluation
Case 39 Patient Note

The following represents a typical note for this patient encounter . the detail may vary depending on the information given by the simulated patient.

History : Describe the history you just obtained form this patient . Include only information (Pertinent positives and negatives)  relevant to this patient¡¦s problem(s).

. 53-yo man with 2 years of episodic midepigastric pain worse with food and sometimes relieved with antacids
. Pain also occurring at night and sometimes radiating to the back
. Pain is 7/10 severity
. Decreased appetite , feeling of a full stomach . abdominal bloating , occasional back stools , and a 6.7-kg(15-lb) weight loss in the past 6 months.

ROS : No jaundice , fever ,chills, vomiting , shortness of breath , diarrhea , or constipation
PMHx : Osteoarthritis of the knee for past 10 years
PSHx : None
Meds : Over-the -counter antacids as needed , ibuprofen 600mg 3 times a day as needed
Allergies : None
FHx : Father healthy , mother died at age 60 of pancreatic cancer
SHx : No  tobacco use ; 2 beers day for 25 years

Physical examinations : Describe any positive and  negative findings relevant  to this patient¡¦s problem(s) . be careful to include only those parts of the examination performed in this encounter.

. Vital signs : Temperature , 36.1C (97F) ; blood pressure , 120/70 mmHg; pulse , 84/min; and respirations ,16/min
. Neck : Supple without thyromegaly or lymphadenopathy
. Heart : RRR with no murmurs
. Lungs : Clear to auscultation and percussion
. Abdomen : Non0tender , non-distended , normative bowel sounds throughout , no hepatosplenomegaly , no bruits

Data interpretation : Based on what you have learned from the history and physical examination, list up to 3 diagnosis that might explain this patient¡¦s complaint(s) . List your diagnoses from most to least likely . For some cases , fewer than 3 diagnosis will be appropriate . the , enter the positive and negative findings form the history and the physical examination (if present) that support each diagnosis . Finally , list initial diagnostic (if any) you would order for each listed diagnosis (eg,restricted physical examination maneuvers, laboratory tests , imaging ECG , etc).

Diagnosis #1 : Peptic ulcer disease

History finding(s)
. Midepigastric pain
. Relief with antacids
. History of NSAID use

Physical examination finding(s)
. None

Diagnosis #2 : Chronic pancreatitis

History finding(s)
. Chronic midepigastric pain
. Pina radiating to back
. History of alcohol use

Physical examination finding(s)
. None

Diagnosis #3 : Gastric cancer

History finding(s)
. Midepigastric pain increased with food
. Nocturnal pain
. Weight loss

Physical examination finding(s)
. None

Diagnostic studies

. Abdominal X-ray (is normal)
. CBC with differential
. Serum amylase and lipase
. Upper GI endoscopy
. Liver function tests








µoªí¤å³¹®É¶¡2018/09/13 08:02am¡@IP: ¤w³]©w«O±K[¥»¤å¦@ 7937 ¦ì¤¸²Õ]¡@ 
JuanFe 

 

¸ê®Æ: ¦¹·|­û¥Ø«e¤£¦b½u¤W Male ¨°Às ¥Õ¦Ï®y
«Â±æ: 0
¾y¤O: ¾y¤O: 78957
¸gÅç: ¸gÅç: 35986
¨Ó¦Û: ¦t©z¤¤¡@blank
µo¤å: 1118 ½g
ºëµØ: 0 ½g
¦b½u: 47¤Ñ19®É22¤À57¬í
µù¥U: 2013/06/17
Message ¬d¬Ý¡@·j´M¡@³q°T¿ý¡@½Æ»s¡@¤Þ¥Î¡@¦^ÂФ峹¦^ÂС@[²Ä 41 ¼Ó]
 40 Case 40 scenario ( vomiting blood )

Doorway information about patient

The patient is a 45-year-old ma who comes to the emergency department due to vomiting blood

Vital signs

. Temperature : 36.7¡¦C(98F)
. Blood pressure : 100/60 mmHg
. Pulse : 90/min
. Respirations : 18/min

Basic differential diagnosis

. Peptic ulcer
. Esophageal and gastric varices
. Mallory-Wises tear
. Gastritis
. Erosive esophagitis
. Gastric malignancy
. Vascular ectasia

¡X¡X¡X¡X¡X

Case 40 sim.pt. instruction

If the doctor asks you about anything other than these , just say ¡§ no ,¡¨ or provide an answer that a normal patient might give.

You are a 45-tear-old man who comes to the emergency department with bloody vomiting

History of present illness

. Acute one of symptoms 2 hours ago
. Burning epigastric pain (8/10 severity ) radiating t the back , immediately followed by vomiting with cupful of bright blood
. Came to the emergency department following a second , similar episode 30 minutes ago
. Associated symptoms include:
- Dizziness/ lightheadedness
- Dark black stools occasionally in the last month
. History of heartburn for the last 2 years , worse in the last 2 months . Midepigastric pain 3-4 times a week after meals , especially when you also consume coffee or alcohol; symptoms last 10-15 minutes and are relieved with antacids

. Ask the doctor : ¡§ Will I die for this bleeding ? Is it cancer ?¡¨

Review of systems

. No fever or chills
. No weight loss
. No shortness of breath
. No jaundice , diarrhea , or constipation
. No urinary symptoms
. Heavy work stress

Past medical / family / social history

. Hypertension
. Tension headaches
. No surgeries or hospitalization
. Medications ; Hydrochlorothiazide 50mg daily , ibuprofen 400 mg 3 times a day as needed
. No drug allergies
. Father , mother , and 2 siblings are healthy
. Married ,live with wife and 2 children
. Occupation : Sale manager at a marketing company
. Smoking : 2 pack a day for last 25 years
. Alcohol : 2 beers a day for last 25 years
. Recreational drugs : None

Physical examination

HEENT :
. No jaundice or pallor
. Oropharynx clear

Neck :
. Supple without thyromegaly or lymphadenopathy

Lungs :
. Clear to auscultation bilaterally

Heart :
. Regular rhythm and rate
. No murmurs , gallops, or rubs

Abdomen :
. Non-tender , non-distended
. Normative bowel sounds throughout
. No hepatosplenomeagly
. No bruits

¡X¡X¡X¡X¡X

Case 40 sim. pt. checklist

Following the encounter , check which of the following items were performed by the examinee

History of present illness/ review of systems

. Asked about the inset and frequency of vomiting
. Asked about the color of the vomit and quantity of blood vomited
. Asked about any recent/ prior symptoms (eg , heartburn , coughing , retching)
. Asked about associated abdominal pain ( location , radiation , quality , severity , aggravating / relieving factors)
. Asked about other associated symptoms , especially :
- Fever
- Dizziness
- Melena or bright red blood in stool
. Asked about hematuria or any otters unusual bleeding /bruising

Past medical /family/social history

. Asked about similar episodes in the past
. Asked about other medical issues (especially liver problems , stomach ulcers)
. Asked about past hospitalizations and surgeries
. Asked about current medications
. Asked about medication allergies . Asked about family health
. Asked about tobacco , alcohol , and recreational drug use
. Asked about occupation

Examination

. Washed heads before examination
. Examined without gown , not through gown
. Examined eyes for pallor and jaundice
. Examined mouth and pharynx
. Palpated neck and supraclavicular region for lymph nodes
. Examined heart and lungs . Examined abdomen (auscultation , superficial and deep palpation , percussion of liver)
. Examined extremities

Counseling

. Explained physical findings and possible diagnosis
. Explained further workup
. Discussed smoking cessation (briefly)

Communication skills and professional conduct

. Knocked before entering the room
. Introduced self and greeted you warmly
. Used your name to address you
. Paid attention to what you said and maintained good eye contact
. Asked opened questions
. Asked non-leading questions . Asked one question at a time
. Listened to what you said without interrupting me
. Used plain English rather than technical jargon
. Used appropriate transition sentences
. Used appropriate draping techniques
. Summarized the history and explained physical findings
. Expressed empathy and gave appropriate reassurances
. Asked whether you have any concerns/questions

Differential diagnosis

. Gastric ulcer
. Duodenal ulcer
. Gastritis
. Erosive esophagitis
. Gastric malignancy

Diagnostic study/studies

. CBC with differential count
. Serum  electrolytes (Na , K , HCO3 , Cl , BUN , creatinine)
. Coagulation studies (PT, aPTT)
. Upper GI endoscopy
. Liver function tests

¡X¡X¡X¡X¡X

Case 40 clinical summary

Clinical Skills Evaluation
Case 40 Patient Note

The following represents a typical note for this patient encounter . the detail may vary depending on the information given by the simulated patient.

History : Describe the history you just obtained form this patient . Include only information (Pertinent positives and negatives)  relevant to this patient¡¦s problem(s).

. 45-yo man with acute onset of 2 episodes of hematemesis with a cup of bright red blood and dizziness
. Midepigastric pain radiating to the back
. 2 years of heartburn and chronic midepigastric pain after meals , last 15-20 minutes and relieved with antacids.
. Symptoms worse with caffeine and alcohol intake
. Occasional black stools in the past month

ROS : No jaundice , fever, chills , shortness of breath , weight los, urinary symptoms , diarrhea , or constipation
PMHx : HTN , tension headaches
PSHx : None
Meds ; Hydrochlorothiazide 50mg daily , ibuprofen 400 mg 3 times daily as needed
Allergies ; None
FHx : Father , mother , and sibling s are ha;thy
SHx : 2 PPD smoker for 25 years ,2 beers a day for 25 years

Physical examinations : Describe any positive and  negative findings relevant  to this patient¡¦s problem(s) . be careful to include only those parts of the examination performed in this encounter.

. Vital signs : Temperature , 36.7¡¦C (98.1F) ; blood pressure , 100/60mmHg; pulse , 90/min ; and respirations , 18/min
. HEENT : No jaundice or pallor , oropharynx clear
. Neck : Supple without thyromegaly or lymphadenopathy
. Lung s; Clear to auscultation bilaterally
. Heart ; RRR without murmurs , gallops, or rubs
. Abdomen : Non-tender , non distended , normative bowel sounds throughout , no  hepatosplenomegaly , no bruits

Data interpretation : Based on what you have learned from the history and physical examination, list up to 3 diagnosis that might explain this patient¡¦s complaint(s) . List your diagnoses from most to least likely . For some cases , fewer than 3 diagnosis will be appropriate . the , enter the positive and negative findings form the history and the physical examination (if present) that support each diagnosis . Finally , list initial diagnostic (if any) you would order for each listed diagnosis (eg,restricted physical examination maneuvers, laboratory tests , imaging ECG , etc).

Diagnosis #1 : Bleeding gastric ulcer

History finding(s)
. Hematemesis
. Midepigastric pain
. Occasional black stools
. NSAID use

Physical examination finding(s)
. Hypoetsnion

Diagnosis #2 : Gastritis

History finding(s)
. Hematemesis
. Midepigastric pain
. History of NSAID use

Physical examination finding(s)
. None

Diagnosis #3 : Esophagitis

History finding(s)
. History of heartburn
. Hematemesis
. Chronic tobacco / alcohol use

Physical examination finding(s)
. None

Diagnostic studies

. CBC with differential
. Upper GI endoscopy
. PT, aPTT
. Basic metabolic panel








µoªí¤å³¹®É¶¡2018/09/13 08:04am¡@IP: ¤w³]©w«O±K[¥»¤å¦@ 8350 ¦ì¤¸²Õ]¡@ 
JuanFe 

 

¸ê®Æ: ¦¹·|­û¥Ø«e¤£¦b½u¤W Male ¨°Às ¥Õ¦Ï®y
«Â±æ: 0
¾y¤O: ¾y¤O: 78957
¸gÅç: ¸gÅç: 35986
¨Ó¦Û: ¦t©z¤¤¡@blank
µo¤å: 1118 ½g
ºëµØ: 0 ½g
¦b½u: 47¤Ñ19®É22¤À57¬í
µù¥U: 2013/06/17
Message ¬d¬Ý¡@·j´M¡@³q°T¿ý¡@½Æ»s¡@¤Þ¥Î¡@¦^ÂФ峹¦^ÂС@[²Ä 42 ¼Ó]
 41 Case 41 scenario ( dizziness )

Doorway information about patient

The patient is a 60-year-old man who comes to the office due to dizziness

Vital signs

. Temperature : 36.1¡¦C (97F)
. Blood pressure : 140/90 mmHg
. Pulse ; 80 /min
. Respirations : 16 /min

Basic differential diagnosis

. Vertigo (benign paroxysmal positional vertigo , labyrinthitis , Meziere disease , posterior circulation stroke )
. Lightheartedness (hypotension ,postural hypotension , arrhythmia , anemia, congestive heart failure)
. Disequilibrium (anxiety , autonomic  dysfunction)
. TIA or stroke
. Medication affect
. Hypoglycemia

¡X¡X¡X¡X¡X

Case 41 sim.pt. instruction

If the doctor asks you about anything other than these , just say ¡§ no ,¡¨ or provide an answer that a normal patient might give.

You are a 60-year-old man who has dizziness

History of present illness

. Onset 4 days ago
. Intermittent espies latina 2-5 minutes
. Worse when standing up form a seated position ; better when lying down
. Sometimes associated with palpitations and sweating
. No associated loss of consciousness
. During an espies earlier today , you noticed weakness in the right lower leg that lasted 10 minutes and resolved spontaneously

. Asked this questions: ¡§ Doc , did I have a stroke? ¡§
. If the examinee asked you to clarify the quality of dizziness , say: ¡§ If feels alike I might fall over or pass out.¡¨

Review of systems

. No headaches
. No changes in hearing or ringing in the ear
. No nausea , vomiting , or diarrhea . No chest pain
. No incontinence of bowel or bladder
. Decreased force of urine stream with nocturia 1 or 2 times a night

Past medical / family / social history

. Type 2 diabetes : Diagnosed 15 years ago and treated with glyburide twice daily and atovastatin ; finger -stick blood glucose has recently been 120-160 mg/dl¡¦
. Hypertension : Diagnosed 10 years ago ; changed 1week ago form atenolol (which you had taken for many years ) to terazosin due to increasing symptoms of benign prostatic hyperplasia
. No history of heart dies or stroke
. No other medications
. No drug allergies
. No surgeries or hospitalizations
. Father and motor died in their 80s due to chronic obstructive pulmonary disease
. Married , live with wife
. Occupation : Supermarket cashier
. Tobacco : 2 packs a day for last 30 years
. Alcohol : No
. Recreational drugs : No

Physical examination

HEENT :
. Cerumen accumulation in both ears
. Normal nares , mouth ,and pharynx

Neck : . No goiter or palpable lymph modes

Cardiovascular :
. Regular rate and rhythm
. No murmur
. No carotid bruits
. Pedal pulses faint but palpable

Neurologic :
. Gait slow and hesitant
. Cranial nerves normal . Normal balance and coordination
. Normal strength and reflexes

If the examinee performs the Dix-Hallpike maneuver (lie back quickly with head turned 45 degrees ), there are no eye movements elicited but say :¡¨ I feel kind of dizzy.¡¨

¡X¡X¡X¡X¡X

Case 41 sim. pt. checklist

Following the encounter , check which of the following items were performed by the examinee

History of present illness/ review of systems

. Asked to clarify the quality of dizziness ( eg, spinning/vertigo , lightheaded)
. Asked about the onset and duration of symptoms
. Asked about the relation to posture and otters triggering factors ; asked about any relieving factors
. Asked whether dizziness is continuous or intermittent
. Asked about associated symptoms (eg, visual changes, headaches , tingling /numbness, weakness , loss of consciousness)
. Asked about ear problems (eg , loss of hearing , ringing in ears)
. Asked about nausea and vomiting

Past medical /family/social history

. Asked about similar episodes in the past
. Asked about past medical issues , hospitalizations ,and surgeries
. Asked about current medications
. Asked about medication allergies
. Asked about family health
. Asked bout tobacco , alcohol , and recreational drug use
. Asked about current living situation and family support
. Asked about occupation

Examination

. Washed heads before examination
. Examined without gown , not through gown
. Checked BP on both arms in sitting , standing , and lying positions
. Examined ears , nose , and throat
. Checked motor power and sensation in all limbs
. Checked reflexes
. Examined cranial nerves
. Performed Romberg test and coordination / cerebellar function tests (eg, finger-to-nose)
. Checked gate
. Performed Dix-Hallpike maneuver (patient lies down quickly with head turned 45 degrees to each side)
. Examined heart , carotid arteries , and distal circulation

Counseling

. Explained physical findings and possible diagnosis
. Explained further workup
. Discussed smoking cessation and readiness to quit

Communication skills and professional conduct

. Knocked before entering the room
. Introduced self and greeted you warmly
. Used your name to address you
. Paid attention to what you said and maintained good eye contact
. Asked opened questions
. Asked non-leading questions . Asked one question at a time
. Listened to what you said without interrupting me
. Used plain English rather than technical jargon
. Used appropriate transition sentences
. Used appropriate draping techniques
. Summarized the history and explained physical findings
. Expressed empathy and gave appropriate reassurances
. Asked whether you have any concerns/questions

Differential diagnosis

. Drug-induced postural hypotension
. Benign paroxysmal positional vertigo
. Autonomic dysfunction from disease
. Hypoglycemia
. Arrhythmia
. Transient ischemic attack

Diagnostic study/studies

. CBC
. Doppler carotid study
. Blood glucose and hemoglobin A1c
. Serum electrolytes (Na, K, Cl, BUN, creatinine)
. ECG
. Head CT

¡X¡X¡X¡X¡X

Case 41 clinical summary

Clinical Skills Evaluation
Case  Patient Note

The following represents a typical note for this patient encounter . the detail may vary depending on the information given by the simulated patient.

History : Describe the history you just obtained form this patient . Include only information (Pertinent positives and negatives)  relevant to this patient¡¦s problem(s).

. 60-yo man with 4 days of intermittent dizziness
. Brief episodes of nonspecific  dizziness triggered by positional changes
. Associated with palpitations and 1 episode of leg weakness (resolved spontaneously) but no loss of consciousness

ROS : No headache , hearing symptoms , nausea , chest pain , or incontinence ; positive for lower urinary tract symptoms
PMHx : HTN, diabetes , BPH
PSHx : None
Meds ; Glyburide , atovastatin , terazosin (chenaged from atenolol a week ago)
Allergies : None
FHx : Father and mother died of COPD
SHx : Married , works as casher , 2 PPD smoker for 30 years , no alcohol or drug use

Physical examinations : Describe any positive and  negative findings relevant  to this patient¡¦s problem(s) . be careful to include only those parts of the examination performed in this encounter.

. vital signs : Temperature , 36.1¡¦C(97F) ; blood pressure , 140/90 mmHg ; pulse , 80/min; and respirations , 16/min
. HEENT : Cerumen accumulation bilaterally ; ears , nose , and throat otherwise normal
. Neck : No lymphadenopathy or thyromegaly
. Heart : RRR with no murmurs , no carotid bouts , pedal pulses palpable but diminished
. Neurologic : Slow gait , CN II-XII normal , normal motor strength and reflexes , Romberg and finger-to-nose tests normal , Dix-Hallpike test subjectively positive

Data interpretation : Based on what you have learned from the history and physical examination, list up to 3 diagnosis that might explain this patient¡¦s complaint(s) . List your diagnoses from most to least likely . For some cases , fewer than 3 diagnosis will be appropriate . the , enter the positive and negative findings form the history and the physical examination (if present) that support each diagnosis . Finally , list initial diagnostic (if any) you would order for each listed diagnosis (eg,restricted physical examination maneuvers, laboratory tests , imaging ECG , etc).

Diagnosis #1 : Benign paroxysmal positional vertigo

History finding(s)
. Intermittent / episodic dizziness
. Symptoms provoked by position change

Physical examination finding(s)
. Subjectively positive Dix-Hallpike test

Diagnosis #2 : Medication -induced postural hypotension

History finding(s)
. Symptoms trigged by position changes
. Recent initiation of alpha blocker

Physical examination finding(s)
. None

Diagnosis #3 : Transient ischemic attack

History finding(s)
. Chronic diabetes and hypertension
. Unilateral lower extremity weakness that resolved spontaneously

Physical examination finding(s)
. None

Diagnostic studies

. ECG
. Electrolytes and blood glucose
. CT scan of brain








µoªí¤å³¹®É¶¡2018/09/13 08:04am¡@IP: ¤w³]©w«O±K[¥»¤å¦@ 9220 ¦ì¤¸²Õ]¡@ 
JuanFe 

 

¸ê®Æ: ¦¹·|­û¥Ø«e¤£¦b½u¤W Male ¨°Às ¥Õ¦Ï®y
«Â±æ: 0
¾y¤O: ¾y¤O: 78957
¸gÅç: ¸gÅç: 35986
¨Ó¦Û: ¦t©z¤¤¡@blank
µo¤å: 1118 ½g
ºëµØ: 0 ½g
¦b½u: 47¤Ñ19®É22¤À57¬í
µù¥U: 2013/06/17
Message ¬d¬Ý¡@·j´M¡@³q°T¿ý¡@½Æ»s¡@¤Þ¥Î¡@¦^ÂФ峹¦^ÂС@[²Ä 43 ¼Ó]
 42 Case 42 scenario ( new-onset seizure )

Doorway information about patient

The patient is a 30-year-old man who comes to the emergency department due to new-onset seizure

Vital signs

. Temperature : 37.2¡¦C (99F)
. Blood pressure : 120/80 mmHg
. Pulse ; 82/min, regular
. Respirations : 18/min

Basic differential diagnosis

. Seizes (secondary to head trauma , infections , drugs , metabolic disorders)
. Hypoglycemia
. Syncope
. Migraine
. Stroke
. Psychogenic seizure
. Space-occupying lesion
. Alcohol or drug withdrawal

¡X¡X¡X¡X¡X

Case 42 sim.pt. instruction

If the doctor asks you about anything other than these , just say ¡§ no ,¡¨ or provide an answer that a normal patient might give.

You are a 30-year-old man who is brought to the emergency department with a new seizure

History of present illness

. Seizure occurred a few hours ago
. Witnesses (coworkers) noted shaking lasting about 3 minutes , followed by 20 minutes of loss of consciousness
. Symptoms preceded by nausea
. Bit tongue but did not pass urine  or feces during the episode
. Have noticed some weakness in the right hand for the past 3 months
. Chronic, occasional mild headaches , but recently the headaches are constant and more severe
. No history of head trauma

Review of systems

. Mild fever ; cold and flu-like symptoms for the past couple of days
. No ear discharge or sinus pain
. No neck pain

Past medical / family / social history

. Type 1 diabetes diagnosis 15 years ago and treated with insulin pump (If the examinee asks you if you think this could be due to hypoglycemia , say: ¡§ I don¡¦t think so because I know what that looks like.¡¦ )
. No other medical issues , hospitalizations , or surgeries
. No otters medications
. No drug allergies
. Father , mother , and sister are healthy ( No family history of seizures)
. Single , not sexually active
. Occupation : Postal clerk
. Smoking : No
. Alcohol : Social occasions only ; last drink was 2 days ago
. Recreational drugs : No

Physical examination

HEENT :
. Normal , no injuries

Neck :
. Supple with no goiter or lymphadenopathy

Heart :
. regular rate and rhythm
. No murmurs

Chest / lungs :
. Clear to auscultation and percussion

Neurologic :
. Awake and alert
. Cranial nerve examination normal
. Motor strength and reflexes normal

¡X¡X¡X¡X¡X

Case 42 sim. pt. checklist

Following the encounter , check which of the following items were performed by the examinee

History of present illness/ review of systems

. Asked bout a description of the seizure
- Duration
- Shaking
- Automatisms (eg, lip smaking)
- Fecal/urinary incontinence
- Biting of tongue or other injuries
- Post-seizure confusion /loss of consciousness
. Asked about aura (prodromal symptoms) and activities at the onset of seizures
. Asked about any recent head trauma
. Asked about associated symptoms:
- Palpitations , chest pain
- Headaches
- Nausea / vomiting
- Fever
- Muscle weakness
. Asked about any past seizures of loss of consciousness

Past medical /family/social history

. Asked about other medical issues(especially diabetes , meningitis /encephalitis, neurologic disorders)
. Asked about prior hospitalizations and surgeries
. Asked about current medications
. Asked bout medication allergies
. Asked about family health
. Asked about tobacco , alcohol , and recreational drug use ( including most recent alcohol use and any history of alcohol withdrawal)
. Asked about occupation

Examination

. Washed heads before examination
. Examined without gown , not through gown
. Examined for head injury and neck stiffness
. Checked motor power , reflexes, and sensation in all limbs
. Examined the cranial nerves
. Examined eyes with ophthalmoscope
. Examined ears with otoscope
. Examined heart and lungs
. Examined abdomen

Counseling

. Explained physical findings and possible diagnosis
. Explained further workup
. Discussed family support

Communication skills and professional conduct

. Knocked before entering the room
. Introduced self and greeted you warmly
. Used your name to address you
. Paid attention to what you said and maintained good eye contact
. Asked opened questions
. Asked non-leading questions . Asked one question at a time
. Listened to what you said without interrupting me
. Used plain English rather than technical jargon
. Used appropriate transition sentences
. Used appropriate draping techniques
. Summarized the history and explained physical findings
. Expressed empathy and gave appropriate reassurances
. Asked whether you have any concerns/questions

Differential diagnosis

. Intracranial mass
. Hypoglycemia
. Alcohol withdraw
. Meningitis / encephalitis

Diagnostic study/studies

. CBC with differential
. Serum electrolytes (Na,K, Cl, CO2, BUN, Cr, Ca, Mg) and glucose
. LFTs
. Urinalysis and urine toxicology screen
. Head CT scan
. Lumber puncture
. EEG

¡X¡X¡X¡X¡X

Case 42 clinical summary

Clinical Skills Evaluation
Case 42 Patient Note

The following represents a typical note for this patient encounter . the detail may vary depending on the information given by the simulated patient.

History : Describe the history you just obtained form this patient . Include only information (Pertinent positives and negatives)  relevant to this patient¡¦s problem(s).

. 30-yo man with new-onset seizure
. Onset a few hours ago
. Single episode of shaking (3 min) followed by loss of consciousness (20min)
. Proceeded by nausea and associated with biting of tongue
. Recent history of right hand weakness and increasing headaches

ROS : Mild fever and flu-like illness last few days
PMHx : Type 1 diabetes for 15 years
PSHx : None
Meds : Insulin via pump
Allergies : None
FHx : Father , mother , and sister are healthy
SHx : Single , works as postal clerk ; social alcohol , no tobacco or drug use

Physical examinations : Describe any positive and  negative findings relevant  to this patient¡¦s problem(s) . be careful to include only those parts of the examination performed in this encounter.

. Vital signs ; Temperature , 37.2¡¦C (99F); blood pressure , 120/80mmHg; pulse , 82/min; and respirations , 18/min
. HEENT : PERRLA , EOMI, no papilledema
. Neck ; Supple without thyromegaly or lymphadenopathy
. Heart : RRR with no murmurs
. Lungs : Clear to auscultation and percussion
. Neurologic ; Awake and cranial nerves II-XII intact , motor strength and reflexes normal

Data interpretation : Based on what you have learned from the history and physical examination, list up to 3 diagnosis that might explain this patient¡¦s complaint(s) . List your diagnoses from most to least likely . For some cases , fewer than 3 diagnosis will be appropriate . the , enter the positive and negative findings form the history and the physical examination (if present) that support each diagnosis . Finally , list initial diagnostic (if any) you would order for each listed diagnosis (eg,restricted physical examination maneuvers, laboratory tests , imaging ECG , etc).

Diagnosis #1 : Intracranial mass

History finding(s)
. New seizure
. Increasing headaches
. Recent subjective weakness

Physical examination finding(s)
. None

Diagnosis #2 : Hypoglycemia

History finding(s)
. Type 1 diabetes
. Use of insulin pump

Physical examination finding(s)
. None

Diagnosis #3 : Alcohol withdraw

History finding(s)
. New Seizure
. Last alcohol intake 2 days ago

Physical examination finding(s)
. None

Diagnostic studies

. CT scan of the head
. CBC with differential
. Serum electrolytes and glucose
. Urine toxicology screen
. Lumbar puncture
. EEG







µoªí¤å³¹®É¶¡2018/09/13 08:06am¡@IP: ¤w³]©w«O±K[¥»¤å¦@ 8030 ¦ì¤¸²Õ]¡@ 
JuanFe 

 

¸ê®Æ: ¦¹·|­û¥Ø«e¤£¦b½u¤W Male ¨°Às ¥Õ¦Ï®y
«Â±æ: 0
¾y¤O: ¾y¤O: 78957
¸gÅç: ¸gÅç: 35986
¨Ó¦Û: ¦t©z¤¤¡@blank
µo¤å: 1118 ½g
ºëµØ: 0 ½g
¦b½u: 47¤Ñ19®É22¤À57¬í
µù¥U: 2013/06/17
Message ¬d¬Ý¡@·j´M¡@³q°T¿ý¡@½Æ»s¡@¤Þ¥Î¡@¦^ÂФ峹¦^ÂС@[²Ä 44 ¼Ó]
 43 Case 43 scenario ( rectal bleeding )

Doorway information about patient

The patient is a 23-year-old man who comes to the emergency department due to rectal bleeding

Vital signs
. Temperature : 38.3¡¦C(101F)
. Blood pressure : 110/60 mmHg
. Pulse : 90/min
. Respirations : 18/min

Basic differential diagnosis

Young patients
. Anal fistula /fissure
. Hemorrhoid
. Inflammatory bowel disease
. Infectious colitis
. Neoplasm
. Vascular ectasis

Elderly patients
. Diverticulitis
. Angiodysplasia
. Malignancy / polyp
. Ischemia colitis
. Inflammatory bowel disease

¡X¡X¡X¡X¡X

Case 43 sim.pt. instruction

If the doctor asks you about anything other than these , just say ¡§ no ,¡¨ or provide an answer that a normal patient might give.

You are a 23-year-old man who comes to the emergency department with rectal bleeding

History of present illness

. Symptoms started with small blood speaks in stools for the last month ; have been passing frank blood for the past 3 days
. Chronic constipation for past 5 years . Often have to strain while defecating and sometimes has severe pain with bowel movements . Diet low in fruits and vegetables
. Bowel movements have increased in frequency recently to 3 times a day
. Associated symptoms:
- Crampy , mild pain (2-3/10) in lower abdomen
- No nausea or vomiting ; no black , tarry stools
- Mild fever (37.8¡¦C[100F]) without chills for the past 4-5 days
- No recent change in weight or appetite

Past medical / family / social history

. No similar episodes in the past
. No otters medical issues , surgeries , or hospitalization
. No medications
. No drug allergies
. Father died of colon cancer at age 65; other family history its unknown
. Single , with multiple , recent female sexual partners ( Do not always use condoms) , no history of sex with men
. Occupation: Restaurant manager and bartender
. Smoking : No
. Alcohol : 1-2 drinks a week
. Recreational drugs : No

Physical examination

HEENT :
. No scleral icterus or pallor

Heart : . Regular rate and rhythm
. No murmurs

Lungs :
. Clear to auscultation

Abdomen :
. Soft , Non-tender , no -distended
. Normal bowel sounds

¡X¡X¡X¡X¡X

Case 43 sim. pt. checklist

Following the encounter , check which of the following items were performed by the examinee

History of present illness/ review of systems

. Asked about the description of the bleeding (eg, color [red/purple/black], quantity , blood on feces or mixed in)
. Asked about the onset and course over time
. Asked about pain during defecation and tenuous
. Asked about abdomen pain
. Asked about any prior gastrointestinal bleeding
. Asked about nausea and vomiting
. Asked about usual and recent bowel and dietary habits
. Asked about appetite and changes in weight
. Asked about fever and chills

Past medical /family/social history

. Asked bout otters medical issues
. Asked bout past hospitalizations and surgeries
. Asked about current medications (especially aspirin)
. Asked about medication allergies
. Asked about family health (especially gastrointestinal malignancy)
. Asked out tobacco , alcohol , and recreational drug use
. Asked occupation
. Asked about sexual history

Examination

. Washed heads before examination
. Examined without gown , not through gown
. Examined abdomen (inspection, auscultation , superficial and deep palpation)

Counseling

. Explained the physical finding and possible diagnosis
. Explained further workup
. Discussed the need for rectal examination

Communication skills and professional conduct

. Knocked before entering the room
. Introduced self and greeted you warmly
. Used your name to address you
. Paid attention to what you said and maintained good eye contact
. Asked opened questions
. Asked non-leading questions . Asked one question at a time
. Listened to what you said without interrupting me
. Used plain English rather than technical jargon
. Used appropriate transition sentences
. Used appropriate draping techniques
. Summarized the history and explained physical findings
. Expressed empathy and gave appropriate reassurances
. Asked whether you have any concerns/questions

Differential diagnosis

. Inflammatory
. Anal fistula / fissure
. Hemorrhoid
. Proctitis
. Infectious colitis
. Neoplasm

Diagnostic study/studies

. Rectal examination
. Facal occult blood test
. CBC
. Coagulation studies (PT/aPTT)
. Anoscopy
. Sigmoidoscopy / colonoscopy
. Abdomen x-ray

¡X¡X¡X¡X¡X

Case 43 clinical summary

Clinical Skills Evaluation
Case 43 Patient Note

The following represents a typical note for this patient encounter . the detail may vary depending on the information given by the simulated patient.

History : Describe the history you just obtained form this patient . Include only information (Pertinent positives and negatives)  relevant to this patient¡¦s problem(s).

. 23-yo man with rectal bleeding
. Mild symptoms for 1 month , passing frank blood last 3 days.
. Associated with crampy pain and low-grade fever
. History of chronic constpation

ROS : No change in weight or appetite , no nausea or vomiting
PMHx : Unremarkable
PSHx : None
Meds : None
Allergies : None
FHx : Father died of colon cancer at age 65
SHx : Single , works as restaurant manager and bartender , sexually active with multiple female partners, light alcohol intake , no tobacco or drug use

Physical examinations : Describe any positive and  negative findings relevant  to this patient¡¦s problem(s) . be careful to include only those parts of the examination performed in this encounter.

. Vital signs : Temperature , 38.3¡¦C(100.9F) ; blood pressure , 110/60mmHg ; pulse , 90/min; and respirations , 18/min
. HEENT : No scleral icterus or pallor
. Heart : RRR with no murmur
. Lungs ; Clear to auscultation
. Abdomen : Soft , non-tender, normal bowel sounds

Data interpretation : Based on what you have learned from the history and physical examination, list up to 3 diagnosis that might explain this patient¡¦s complaint(s) . List your diagnoses from most to least likely . For some cases , fewer than 3 diagnosis will be appropriate . the , enter the positive and negative findings form the history and the physical examination (if present) that support each diagnosis . Finally , list initial diagnostic (if any) you would order for each listed diagnosis (eg,restricted physical examination maneuvers, laboratory tests , imaging ECG , etc).

Diagnosis #1 : Hemorrhoid

History finding(s)
. Visible lower gastrointestinal bleeding
. Chronic constipation and low-fiber intake

Physical examination finding(s)
. None

Diagnosis #2 : inflammatory bowel disease

History finding(s)
. Visible lower gastrointestinal bleeding
. Vampy abdominal pain

Physical examination finding(s)
. Fever

Diagnosis #3 : Rectal cancer

History finding(s)
. Visible lower gastrointestinal bleeding
. First-degree family history of colon cancer

Physical examination finding(s)
. None

Diagnostic studies

. Rectal examination
. CBC
. Electrolytes and blood glucose
. PT/ aPTT
. Anoscopy
. X-ray of abdomen







µoªí¤å³¹®É¶¡2018/09/13 08:06am¡@IP: ¤w³]©w«O±K[¥»¤å¦@ 7454 ¦ì¤¸²Õ]¡@ 

 ¦@ 3 ­¶ 9 7 [ 1 2 3 ] 8 :

§Ö³t¦^ÂÐ¥DÃD: Sample cases
±z¥Ø«eªº¨­¥÷¬O¡G ³X«È ¡A­n¨Ï¥Î¨ä¥L·|­û¨­¥÷¡A½Ð¿é¤J·|­û¦WºÙ©M±K½X¡C¥¼µù¥U³X«È½Ð¿é¤Jºô¦W¡A±K½X¯dªÅ¥Õ¡C
¿é¤J·|­û¦WºÙ©M±K½X: ·|­û¦WºÙ: ¨S¦³µù¥U¡H¡@±K½X: §Ñ°O±K½X¡H
¤W¶Çªþ¥ó©Î¹Ï¤ù (³Ì¤j®e¶q 9000KB)
¥Ø«eªþ¥ó:(¦p¤£»Ý­n¬Y­Óªþ¥ó¡A¥u»Ý§R°£¤º®e¤¤ªº¬ÛÃö [UploadFile ...] ¼ÐÅÒ§Y¥i) [§R°£]
¿ï¶µ

¨Ï¥Î LeoBBS ¼ÐÅÒ¡H
Åã¥Ü±zªºÃ±¦WÀÉ¡H
¨Ï¥Îªí±¡²Å¸¹Âà´«¡H
¨Ï¥Î¦r«¬¼Ë¦¡Âà´«¡H

¡@¡@¡@¡@§Ö³t¤Þ¥Î²Ä ¼Ó¼hªº¦^ÂÐ
 ³»ºÝ¡@¥[¨ì"§Úªº³Ì·R" ¥DÃDºÞ²z¡G Á`©T³» ¨ú®øÁ`©T³» °Ï©T³» ¨ú®ø°Ï©T³» ©T³» ¨ú®ø©T³»
¥[­« ¨ú®ø¥[­« ºëµØ ¨ú®øºëµØ ´£¤É Âê©w ¸ÑÂê §R°£ §R°£¦^ÂÐ ²¾°Ê


© ¤¤¤åª©Åv©Ò¦³¡G ¶ð¤º´Óª«¶é¡@ÁcÅ骩Åv©Ò¦³¡G ¼¯¥§ºô
µ{¦¡ª©Åv©Ò¦³¡G ¹p¶Æ¬ì§Þ¡@ µ{¦¡Â½Ä¶¡Gauron¡@ª©¥»¡G LeoBBS X Build041101
 

¥»½×¾Â¨¥½×¯ÂÄݵo¨¥ªÌ­Ó¤H·N¨£¡A»P ¶ð¤º´Óª«¶é ¥ß³õµLÃö